Chuyên đề bất đẳng thức toán học

75
Một số phương pháp chứng minh bất đẳng thức Chuyên đề Toán học Trần Tiến Đạt Lưu Trung Kiên Nguyễn Hồng Đức Đỗ Thị Hồng Vân Lớp 10 Toán Trường THPT chuyên Trần Phú Hải Phòng. Tháng 5/2009

Transcript of Chuyên đề bất đẳng thức toán học

Page 1: Chuyên đề bất đẳng thức toán học

Một số phương pháp chứng minh bất đẳng thức

Chuyên đề Toán học

Trần Tiến Đạt

Lưu Trung Kiên

Nguyễn Hồng Đức

Đỗ Thị Hồng Vân

Lớp 10 Toán

Trường THPT chuyên Trần Phú

Hải Phòng. Tháng 5/2009

Page 2: Chuyên đề bất đẳng thức toán học

Trang 2

Một số phương pháp chứng minh bất đẳng thức

Phương pháp SOS Phương pháp dồn biến

Phương pháp PQR Phương pháp SS

TRẦN TIẾN ĐẠT – LƯU TRUNG KIÊN NGUYỄN HỒNG ĐỨC – ĐỖ THỊ HỒNG VÂN

----------------------***---------------------- Lớp 10 Toán – THPT chuyên Trần Phú

Hải Phòng. Tháng 5/2009

Page 3: Chuyên đề bất đẳng thức toán học

Trang 3

LỜI NÓI ĐẦU ất đẳng thức là 1 vấn đề khá khó và thú vị trong toán học. Trong các kỳ thi học sinh giỏi toán quốc gia, thi Olympic Toán khu vực và quốc

tế, thi Olympic Toán sinh viên giữa các trường đại học và cao đẳng, các bài toán liên quan đến bất đẳng thức cũng hay được đề cập và thường thuộc loại khó hoặc rất khó. Các bài toán về ước lượng và tính giá trị cực trị của các tổng, tích cũng như các bài toán xác định giới hạn của một số biểu thức cho trước thường có mối quan hệ ít nhiều đến các tính toán, ước lượng bất đẳng thức tương ứng.

Trên thế giới mỗi ngày lại có thêm rất nhiều bất đẳng thức thách thức khả năng tư duy và óc sáng tạo của con người. Lý thuyết bất đẳng thức và đặc biệt, các bài tập về bất đẳng thức rất phong phú và cực kỳ đa dạng. Vì vậy mà không có gì là khó hiểu khi bất đẳng thức luôn được xếp vào lớp các bài toán “truyền thống” của hầu hết các cuộc thi toán. Không chỉ có nhiều ứng dụng trong đời sống, bất đẳng thức còn cho chúng ta những cái nhìn mới mẻ, thôi thúc sự tò mò, say mê nơi mỗi con người. Chúng ta được biết tới các bất đẳng thức cổ điển khá nổi tiếng như AM - GM, Cauchy – Schwarz, Nesbit, BĐT hoán vị… Nhưng cùng với sự phát triển của toán học, các bất đẳng thức hiện đai cũng ngày càng trở nên “chặt” hơn đòi hỏi sự ra đời của các phương pháp mới. Nhóm chúng tôi viết chuyên đề này là để trình bày về 3 chứng minh cơ bản cho các bất đăng thức hiện đại : dồn biến, S.O.S, và phương pháp P.Q.R. Đây là 3 phương pháp khá hiệu quả và phổ biến đề chứng minh nhiều bất đẳng thức ngày nay. Bên cạng 3 phương pháp này chúng tôi cũng giới thiệu cả phương pháp S.S – một phương pháp kết hợp của S.O.S và Schur.

Chúng tôi xin chân thành cảm ơn thầy giáo Đoàn Thái Sơn, người đã giao công việc này và cung cấp nhiều tài liệu cho chúng tôi. Trong quá trình làm chuyên đề này, chúng tôi có sử dụng những kiến thức được trình bày trong cuốn sách “Những viên kim cương trong bất đẳng thức toán học” của tác giả Trần Phương, cuốn “Sáng tạo bất đẳng thức” của anh Phạm Kim Hùng, chuyên đề về phương pháp dồn biến của Phan Thành Việt, chuyên đề về bất đẳng thức Schur và phương pháp đổi biến pqr của Võ Thành Văn. Chúng tôi hi vọng chuyên đề này sẽ giúp bạn đọc phần nào trên con đường đến với bất đẳng thức.

Tác giả: Lưu Trung Kiên Cùng nhóm cộng sự

Trẩn Tiến Đạt - Nguyễn Hồng Đức - Đỗ Thị Hồng Vân Công việc của các thành viên

Phần A: Nguyễn Hồng Đức Phần B: Trần Tiến Đạt – Lưu Trung Kiên Phần C: Đỗ Thị Hồng Vân Phần D: Nguyễn Hồng Đức – Trần Tiến Đạt

B

Page 4: Chuyên đề bất đẳng thức toán học

Trang 4

MỤC LỤC LỜI NÓI ĐẦU ............................................................................................ 3

MỤC LỤC .................................................................................................. 4

A PHƯƠNG PHÁP SOS ........................................................................ 6

A.I. Giới thiệu về phương pháp SOS .................................................................. 6

A.I.1 . Lý thuyết ................................................................................................... 6

A.I.2 . Bài tập ...................................................................................................... 7

A.II. Một số ứng dụng quan trọng của phương pháp S.O.S............................ 14

A.III. Phương pháp S.O.S trong bất đẳng thức hoán vị vòng quanh. .......... 16

B PHƯƠNG PHÁP DỒN BIẾN ............................................................ 18

B.I. Giới thiệu về tư tưởng phép dồn biến ....................................................... 18

B.II. Bất đẳng thức đối xứng 3 biến đạt cực trị tại tâm .................................... 18

B.III. Dồn biến bằng kỹ thuật hàm số .............................................................. 25

B.IV. Bất đẳng thức 3 biến cực trị đạt tại biên ............................................... 31

B.V. Định lý dồn biến mạnh S.M.V và bất đẳng thức 4 biến ............................ 34

B.VI. Dồn biến bằng hàm lồi............................................................................. 38

B.VII. Dồn biến không xác định - UMV ............................................................. 42

B.VIII. Dồn biến toàn miền (EMV) ....................................................................... 44

B.VIII.1 . EMV với biên tại 0:.............................................................................. 44

B.VIII.2 . EMV với biến trong tam giác ............................................................... 45

B.IX. Định lí dồn biến tổng quát {GMV – GENERAL MIXING VARIABLES} .. 47

B.X. Bài tập .......................................................................................................... 50

C BẤT ĐẲNG THỨC SCHUR VÀ PHƯƠNG PHÁP BIẾN ĐỔI PQR . 52

C.I. Bất đẳng thức Schur ................................................................................... 52

C.I.1 . Lý thuyết ................................................................................................. 52

C.I.2 . Ví dụ ....................................................................................................... 55

Page 5: Chuyên đề bất đẳng thức toán học

Trang 5

C.II. Phương pháp biến đổi pqr ......................................................................... 59

C.II.1 . Đa thức đối xứng 3 biến ......................................................................... 59

C.II.2 . Xây dựng các bất đẳng thức của pqr ..................................................... 60

C.II.3 . Ví dụ ....................................................................................................... 61

D PHƯƠNG PHÁP SS (SCHUR–S.O.S) ....................................... 67

D.I. Giới thiệu...................................................................................................... 67

D.II. Phương pháp SS với các bất đẳng thức đạt cực trị tại tâm .................... 68

D.III. Phương pháp SS với các bất đẳng thức cực trị không đạt tại tâm..... 71

LỜI KẾT .................................................................................................. 74

Page 6: Chuyên đề bất đẳng thức toán học

MỘT SỐ PHƯƠNG PHÁP CHỨNG MINH BẤT ĐẲNG THỨC PHƯƠNG PHÁP SOS

Trang 6

A PHƯƠNG PHÁP SOS A.I. Giới thiệu về phương pháp SOS

A.I.1 . Lý thuyết Trước tiên chúng ta sẽ xem xét ví dụ sau: CMR: 2 2 2a b c ab ac bc Giải: BĐT cần chứng minh tương đương với :

21 ( ) 0

2a b

Để chứng minh bất đẳng thức trên không khó, nhưng chúng ta chỉ quan tâm bản chất thực sự của cách làm trên. Cách làm trên đã đưa bất đẳng thức thành dạng tổng của các bình phương, lợi dụng tính chất 2 0X X R để chứng minh bất đẳng thức. Đó cũng chính là ý tưởng của phương pháp S.O.S( sum of square) – đưa bất đẳng thức có dạng 0S thành dạng như sau: 2 2 2( ) ( ) ( ) 0S b c S c a S a ba b c

Nếu chưa làm quen với S.O.S bạn sẽ chỉ giải quyết được bài toán khi mà , , 0S S Sa b c .Nhưng S.O.S có thể giải quyết trường hợp mà 1 trong các số , ,S S Sa b c

âm (

và hầu hết các trường hợp đều xảy ra như vậy). Cụ thể chúng ta có: Giả sử S = 2 2 2( ) ( ) ( ) 0S S b c S c a S a ba b c (*)

Nếu , , 0S S Sa b c thì S 0

Nếu a b c và , , 0S S S S Sb b a c b thì S 0

Nếu a b c và , ,2 , 2 0S S S S S Sa c b a c b thì S 0

Nếu a b c và , 0S Sb c , 2 2 0a S b Sb a thì S 0

Nếu 0, 0S S S S S S S S Sa b c a b b c c a thì S 0

Nhìn chung trong phương pháp S.O.S việc chứng minh các điều kiện trên là khá đơn giản nếu đã đưa được BĐT về dạng (*). Phần khó nhất trong phương pháp S.O.S là làm thế nào để đưa bất đẳng thức về dạng (*). Đây chính là thứ tạo ra nét độc đáo của phương pháp S.O.S khi mà bạn sẽ thấy rằng cách biểu diễn dưới dạng (*) không phải là duy nhất. Sau đây là một số công thức cơ bản hay dùng để các bạn thuận tiện hơn khi chứng minh BĐT bằng phương pháp này:

Phần

A

Page 7: Chuyên đề bất đẳng thức toán học

MỘT SỐ PHƯƠNG PHÁP CHỨNG MINH BẤT ĐẲNG THỨC PHƯƠNG PHÁP SOS

Trang 7

A.I.2 . Bài tập Để cụ thể hóa cho điều này chúng ta hãy xem xét các ví dụ sau

Bài A.I.1: Cho a,b,c là các số dương . CMR: 2 2 2

( )( )( ) 4( ) 12a b b c c a ab bc caabc a b c

Chứng minh

22 2

2

23 3

22 2 2

2 22 2 2

2

2

312

1 13 3

cyc

cyc

a b ab a b

a ba bb a aba b ab a b a b a b

a b c ab bc ca a b

a b c a b c a b

2

23 3 3

22 2

2 2

2

22 2

23 2

24 3 2 2

3

8

132

22

32 2

13

1 231 3 24

cyc

cyc cyc

cyc sym

sym sym cyc

cyc sym sym

cyc sym sym

sy

a b b c c a abc a b c

a b c abc a b c

a ba b a b

a b a b

a bab c a c b c

a b ab a b

a a b a b a b

a a b a ab b a b

a b

33

2 24 2 2

1312

m sym cyc sym

cyc cyc cyc

ab a b a

a a b a b a b

Page 8: Chuyên đề bất đẳng thức toán học

MỘT SỐ PHƯƠNG PHÁP CHỨNG MINH BẤT ĐẲNG THỨC PHƯƠNG PHÁP SOS

Trang 8

2 2 2

2 2 2

2 2 2

2 2

2 2 2

2 2 2

( )( )( ) 4( ) 12

( )( )( ) 4( ) 4( )8 0

( ) 2 ( )0

( ) ( ) ( ) 0

( )

cyc cyc

a b b c c a ab bc caabc a b c

a b b c c a ab bc ca a b cabc a b c

a b c b c

abc a b cS b c S c a S a ba b c

Trong đó:

2 2

2 2 2 2 2 2

2 2

2 2 2 2 2 2

2 2

2 2 2 2 2 2

1 2 ( )( )

1 2 ( )( )

1 2 ( )( )

a b cSa bc a b c bc a b cb c aSb ca a b c ac a b cc a bSc ab a b c ab a b c

Dễ thấy: Sa , Sb , Sc đều là các số dương. Vậy bất đẳng thức đã được chứng minh. Dấu bằng

xảy ra khi a b c

Bài A.I.2: Cho a,b,c là các số dương. CMR 2 3 3 3

2 2 2

3( )sym

a a b cb a b c

Chứng minh

2 3 3 3

2 2 2

2 3 3 3

2 2 2

2 22 2

2 2 2

22

2 2 2

2 2 2

3( )

3( )( )

( ) ( )2

( )

( ) ( )( )

( ) ( ) ( ) 0

( )

sym

sym cyc cyc

cyc

sym

sym sym

a a b cb a b c

a a b ca ab a b c

a a b b ca ab b

b a b c

a b a ba bb a b c

S b c S c a S a ba b c

trong đó:

2 2 2

2 2

2 2

a

b

c

b a cSc

b c abSa

a c abSb

Trường hợp 1: a b c Dễ thấy Sa , Sc > 0. Ta cần CM: 2 , 2 0a c a bS S S S

Page 9: Chuyên đề bất đẳng thức toán học

MỘT SỐ PHƯƠNG PHÁP CHỨNG MINH BẤT ĐẲNG THỨC PHƯƠNG PHÁP SOS

Trang 9

Xét 3 3 3

3

2 2 2

3

22 3

2 2 32

2 3

543 2 0( )

( ) ( ) 6 ( )0

2 ( )

1 1 1 6( ) ( 02 4 ( ) 4 ( )

2 ( ) ( ) ( ) 24( ) ( ) 02 4 ( ) 4 ( )

a b c abcabc a b c

a b c a b c a b cabc a

b c ab cabc bc a b c bc a b c

b c a a b c b c a b c abcb cabc bc a b c bc a b c

mà 3 2 2 32 2 ( )a b c b a c abc a b c

2 0a bS S

Xét 2 2 3 2 2 3( ) ( 2 ) 22a c

a b c b b a c abc a c cS Sbc

mà 3 2 2 32 2 ( )b a c a b c abc a b c

2 0a cS S

Trường hợp 2: c b a Dễ thấy , 0a cS S . Ta cần CM: 2 , 2 0a c a bS S S S

Bằng cách làm như trên ta cũng sẽ chứng minh được 2 , 2 0a c a bS S S S Vậy bất đẳng thức đã được chứng minh. Dấu bằng xảy ra khi a b c

Bài A.I.3: Cho a,b,c là các số dương. CMR: 3 3 3

3

54 5( )

a b c abcabc a b c

Chứng minh

Bất đẳng thức tương đương với 3 3 3

3

2 2 2

3

22 3

2 2 32

2 3

543 2 0( )

( ) ( ) 6 ( )0

2 ( )1 1 1 6( ) ( 0

2 4 ( ) 4 ( )2 ( ) ( ) ( ) 24( ) ( ) 0

2 4 ( ) 4 ( )

a b c abcabc a b c

a b c a b c a b cabc a

b c ab cabc bc a b c bc a b c

b c a a b c b c a b c abcb cabc bc a b c bc a b c

(luôn đúng). Vậy bất đẳng thức đã được chứng minh. Dấu bằng xảy ra khi a b c

Page 10: Chuyên đề bất đẳng thức toán học

MỘT SỐ PHƯƠNG PHÁP CHỨNG MINH BẤT ĐẲNG THỨC PHƯƠNG PHÁP SOS

Trang 10

Bài A.I.4: Cho , , 0a b c .CMR

22

2 3cyc

a b cab c ab bc ca

Chứng minh

Ta có:

22

22

2

2

2 3

2 22 3

14 2 3

32 24 4 2 3

cyc

cyc

cyc

sym

a b cab c ab bc ca

a b ca a b c a b cb c ab bc ca

a b c a b c a b cb c ab bc ca

a b c ab bc caa b c b a c a b ca bb c c a ab bc ca

a b

2 32 2 2

4 2 3 3sym

a b c ab bc caa b c a b cb c c a ab bc ca a b c ab bc ca

2 2

2

12 3 3

1 12 3 3

sym sym

sym

a b a bb c c a ab bc ca a b c ab bc ca

a bb c c a ab bc ca a b c ab bc ca

2 0asym

S b c

trong đó:

1 1

2 3 3

1 12 3 3

1 1

2 3 3

a

b

c

Sa b c a ab bc ca a b c ab bc ca

Sa b b c ab bc ca a b c ab bc ca

Sb c c a ab bc ca a b c ab bc ca

Dễ thấy , 0b cS S

Ta sẽ chứng minh 0a bS S (theo tiêu chuẩn 2). Thật vậy:

1 1 1

3 3a bS S

a b c a a b b c ab bc ca a b c ab bc ca

Page 11: Chuyên đề bất đẳng thức toán học

MỘT SỐ PHƯƠNG PHÁP CHỨNG MINH BẤT ĐẲNG THỨC PHƯƠNG PHÁP SOS

Trang 11

223 3

1 03 3

a b

ab bc ca ab bc ca ab bc ca a b c b

a b c a a b b c ab bc caS

a b c ab b cS

c a

Vậy bất đẳng thức đã được chứng minh. Dấu bằng xảy ra khi a b c

Bài A.I.5: Cho a,b,c là các số dương . CMR: 2 2 22 2 2 3

2

a b ca b ca b b c c a

Chứng minh

Bất đẳng thức cần chứng minh tương đương với:

2 2 22 2 2 3

2 2 2

a b ca b c a b c a b ca b b c c a

(1)

Xét vế trái (1)

2 2 2

2

2 2 2 3

2 2 2 3

2

2

8 8

2 2

4

sym cyc

cyc cyc cyc cyc cyc

cyc cyc cyc

a b c a b ca b b c c a

a b c c a a b b c c a a

a b b c c a a b b c c a

a bc a b a b abc a a b b c c a abc a

a b b c c a a b b c c a a b b c c a

a bc a b a babc a b

a b b c c a

2

22 2 3

2 22 2

2 2

2

3

2

3 3

2

2 2

cyc

cyc cyc cyc

cyc cyc cyc

cyc

a b c a b cc

a b b c c a a b b c c a

a b a b abc a b c a b c a b c

a b b c c a a b b c c a

a b abc a b c ab a b a b c a b c

a b b c c a a b b c c a a b b c c a

a b abc a b c

a b b c c a

2 2

32 2

cyc cycbc b c a b c a b c

a b b c c a a b b c c a

2 2 22

22

3 2

2 2 2

3 22

cyc cyc cyc

sym

a b c bc b c a b c a b c

a b b c c a a b b c c a a b b c c a

a bc a b c ab c

a b b c c a

Xét vế phải (1)

Page 12: Chuyên đề bất đẳng thức toán học

MỘT SỐ PHƯƠNG PHÁP CHỨNG MINH BẤT ĐẲNG THỨC PHƯƠNG PHÁP SOS

Trang 12

2 2 2

2 2 2

22 2 2

2 2 2

2

2 2 2

3

2 2

3

23

2 3

2 3cyc

a b c a b c

a b c a b c

a b c a b c

a b c a b c

b c

a b c a b c

Vậy 1

222

2 2 2

22

2 2 2

2

3 22 2 3

3 2 1 02 2 3

0

sym cyc

sym

acyc

a bc a b c a b cb c

a b b c c a a b c a b c

a bc a b c ab c

a b b c c a a b c a b c

S b c

Trong đó:

2

2 2 2

2

2 2 2

2

2 2 2

2 2 1

3

2 2 1

3

2 2 1

3

a

b

c

a bc ab acSa b b c c a a b c a b c

b ca ab bcSa b b c c a a b c a b c

c ab bc acSa b b c c a a b c a b c

Ta dễ dàng chứng minh được , 0a cS S

Theo tiêu chuẩn 3 ta cần chứng minh 2 0 & 2 0a b c bS S S S (Phần chứng minh này đơn giản, xin dành để các bạn tự chứng minh)

Bài A.I.6: Cho a,b,c là các số dương .CMR:

2 2 22 2 2 6 a b ca b c a b cb c a a b c

Chứng minh

Bất đẳng thức càn chứng minh tương đương với:

Page 13: Chuyên đề bất đẳng thức toán học

MỘT SỐ PHƯƠNG PHÁP CHỨNG MINH BẤT ĐẲNG THỨC PHƯƠNG PHÁP SOS

Trang 13

2 2 22

2 2 2 2

2 2

2

2

2

2

62

2 3

2

1 2 0

0

0

0

sym

sym

sym cyc

sym

sym

sym

acyc

a b ca a a b cb a b c

a b a b c a b cb a b c

a b a bb a b c

a bb a b c

a c ba bb a b c

a c ba bb

S b c

trong đó

a

b

c

a b cSc

b c aSa

a c bSb

2 2 2 2 2 2

3 6 3 3

0

0

a b

a b ccyc

a b b a

b c c a

c c

a bS S Sb a

a b c ab a c b ac b a a bcS S S S S S

abcba bc abc ab ac abc ca cb abc

S S S S S Sabc

theo

tiêu chuẩn 5 bất đẳng thức đã được chứng minh. Dấu bằng xảy ra khi a b c

Có thể nói trong 5 điều kiện của S.O.S thì điều kiện 5 là khó áp dụng nhất, đặc biệt là khi những hệ số , ,a b cS S S phức tạp. Tuy nhiên trong bài toán vừa rồi áp dụng điều kiện 5 là hợp lý nhất (các điều kiện còn lại áp dụng sẽ rất phức tạp và có thể không làm được). Vì vậy một điều hết sức cần thiết là chúng ta phải áp dụng 5 điều kiện cơ bản một cách thật linh hoạt, hợp lí. Có như vậy bạn mới có thể sử dụng được toàn bộ sức mạnh của S.O.S

Bài A.I.7: (IMO 2005 ) Cho a,b,c > 0 thỏa mãn 1abc CMR: 5 2 5 2 5 2

5 2 2 5 2 2 5 2 2 0a a b b c ca b c b c a c a b

(*)

Chứng minh

Page 14: Chuyên đề bất đẳng thức toán học

MỘT SỐ PHƯƠNG PHÁP CHỨNG MINH BẤT ĐẲNG THỨC PHƯƠNG PHÁP SOS

Trang 14

Các phương pháp CM bất đẳng thức hiện đại ( đặc biệt là Schur, PQR, S.O.S) đều rất xem trọng việc đồng bậc hóa bất đẳng thức( qua các ví dụ trên hẳn các bạn đều hiểu được tầm quan trọng của cộng việc này).

Việc đồng bậc hóa (*) được thực hiện thông qua đánh giá sau đây

4 2 2 25 2 5 2 4 2

25 2 2 5 2 2 4 2 2 4 2 2

2

2

a a b ca a a a abc a a bca b c a b c abc a bc b c a b c

Đánh giá trên có thể xem như chìa khóa của bài toán này bởi vì phần CM còn lại rất đơn giản :

Đặt 2 2 2, ,x a y b z c

2 2 2 22

2 2 22 2 2

2 2 2

2, ,

2 2 2

( ) ( ) ( ) 0

cyc cyc

x x y z z z x y x xy yf x y z x y

x y z x y z y x z

S y z S z x S x yx y z

A.II. Một số ứng dụng quan trọng của phương pháp S.O.S

Một ứng dụng quan trọng của S.O.S là sáng tạo ra các bất đẳng thức mới. Đó thường là những bất đẳng thức rất mạnh, khó chứng minh bằng các bất đẳng thức thông thường( Cauchy, BCS...).Sau đây là một số ví dụ quan trọng và điển hình nhất.

Ví dụ A.II.1: Tìm hằng số k tốt nhất để BĐT sau đúng với mọi a,b,c dương:

2 2 2

32

a b c ab bc cak kb c c a a b a b c

Sự khác biệt lớn nhất giữa đẳng thức và bất đẳng thức là dấu , . Điều này thể hiện

rõ khi bạn chứng minh bài toán trên bằng phương pháp thông thường: ‘xử lí’ 2 2 2

ab bc caa b c

dấu Điều bất ngờ là S.O.S lại giải quyết điều này rất dễ dàng. Chứng minh

Bất đẳng thức cần chứng minh tương đương với:

2 2 2

2 2

2 2 2

2 2 22

2 2 2

1 12

1 12 2

0

( ) ( ) ( ) 0

cyc

cyc cyc

cyc

a ab bc cakb c a b c

a b a bk

b c a c a b c

a b ca b kb c a c

S b c S c a S a ba b c

Page 15: Chuyên đề bất đẳng thức toán học

MỘT SỐ PHƯƠNG PHÁP CHỨNG MINH BẤT ĐẲNG THỨC PHƯƠNG PHÁP SOS

Trang 15

trong đó

2 2 2

2 2 2

2 2 2

a

b

a

a b cS ka b a c

a b cS ka b b c

a b cS kc b a c

Điều kiện cần: Cho c b a ta có

2 22

2

2 2 2

2

2

2 02

22 2, ( )

2 2 22 1

2 2 2

2 2 2 0

a ba b kb a b

aa b t abk u a b t

ab a b t bb

t u t

f t t ut u

0f t có nghiệm

2 3 1' 2 2 02

u u u

Điều kiện đủ:

Ta chứng minh 3 12

k là hắng số tốt nhất của bất đẳng thức trên.

2 2 2

2 0cyc

a b ca b kb c a c

(1)

Không mất tính tổng quát giả sử a b c a b cS S S

2 2 2 2 2 2

2a ba b c a b cS S ka b a c a b b c

đặt 2

a bv

22

2 2 2 2

2

2 22 2 22 2 22

2

a b

a bc v c

v c v c v cS S k k kv b c a c v v cb c c a

v

2 222 2 , 0

2a bv cS S k u v c kv v c

(do 3 1,2

u v c k )

Do a b cS S S và 0a bS S nên 0, 0, 0b a b c bS S S S S .

Page 16: Chuyên đề bất đẳng thức toán học

MỘT SỐ PHƯƠNG PHÁP CHỨNG MINH BẤT ĐẲNG THỨC PHƯƠNG PHÁP SOS

Trang 16

Theo tiêu chuẩn 2 suy ra 3 12

k thì (1) đúng với mọi , , 0a b c

Kết luận: 3 12

k là hằng số tốt nhất để bất đẳng thức

2 2 2

32

a b c ab bc cak kb c c a a b a b c

đúng với mọi , , 0a b c

Một câu hỏi muôn thuở: thế nào là một cách giải hay cho một bài toán? Một bài toán có thể có nhiều cách giải, mỗi cách giải mang theo một màu sắc riêng, một nét độc đáo riêng. Chính vì thế khó có thể có một câu trả lời chính xác cho câu hỏi trên vì mỗi người quan niệm cái hay một cách khác nhau (một lời giải hay là một lời giải ngắn nhưng cũng có thể là một lời giải là một sự sáng tạo độc đáo, mang theo một ý tưởng mới, tư tưởng mới). Theo những tiêu chi trên thì lời giải cho bài toán trên chưa phải là một lời hay. Mặc dù vậy nhưng lời giải trên mang tính hiệu quả.

Tìm hằng số k tốt nhất để bất đẳng thức sau đúng:

2 2 21 1 1 9ab bc caa b c k ka b c a b c

Rõ ràng theo phương pháp S.O.S, hai bài toán này giống nhau về mặt bản chất. Để nghĩ ra một cách làm độc đáo cho 2 bài trên là chuyện không dễ chứ đừng nói là tìm một lời giải hay áp dụng cho tất cả các bài toán có dạng trên. Nhưng khi mà bất đẳng thức phát triển nhanh với tốc độ chóng mặt như hiện nay, thì những phương pháp cổ điển hẳn chưa làm hài lòng những người yêu thích bất đẳng thức. Có thể S.O.S không mang lại một lời giải đẹp mắt như BĐT Cauchy hay các phương pháp chứng minh BĐT cổ điển khác, nhưng nó giải được một lớp lớn các bài toán hay và khó, đó cũng chính là ứng dụng lớn lao mà S.O.S mang đến cho chúng ta

A.III. Phương pháp S.O.S trong bất đẳng thức hoán vị vòng quanh.

Chứng minh bất đẳng thức hoán vị vòng quanh luôn khó hơn bất đẳng thức đối xứng. Nhìn chung khi chứng minh BĐT hoán vị chúng ra thường giả sử ax , ,a m a b c hay nói

cách khác là xét 2 trường hợp &a b c a c b . Còn trong phương pháp SOS, chúng ta sẽ xét 2 trường hợp &a b c c b a . Chỉ cần nhớ kĩ 5 điều kiện của S.O.S, chúng ta đều dễ dàng lý giải điiều này.

Ví dụ A.III.1: Cho a,b,c là 3 cạnh tam giác CMR 3 2 3a b c a c bb c a c b a

Phương pháp giải của bài này( và nhiều bài BĐT hoán vị khác ) là quy đồng mẫu số đưa về dạng đồng bậc. Rất đơn giản nhưng hiệu quả thì thật không ngờ!

Chứng minh Bất đẳng thức cần cứng minh tương đương với:

Page 17: Chuyên đề bất đẳng thức toán học

MỘT SỐ PHƯƠNG PHÁP CHỨNG MINH BẤT ĐẲNG THỨC PHƯƠNG PHÁP SOS

Trang 17

2 2 2 2 2 2

3 3 3 2 2 2 3 3 3 2 2 2 3 3 3

2 2 2

2 2 2

3 2 3

3 2 3

2 12 23 2

( ) ( ) ( ) 0sym sym cyc

a c b a c b b c c a a b abc

a b c a c b a c b a b c b c c a a b a b c abc

b a a b a b a b a b c a b

S b c S c a S a ba b c

trong đó 5 5 35 5 35 5 3

a

b

c

S b c aS c a bS a b c

Mặc dù các hệ số , ,a b cS S S khá đơn giản nhưng chúng ta không thể chỉ dùng 5 điều kiện cơ bản để làm được bài toán này. Bổ đề: Nếu

22 1 0a c bS S S

0 01 10 0

c b a b

c b a b

a b c b c b a bS S S S

S S S S

thì 0S Áp dụng: Không mất tính tổng quát giả sử b nằm giữa a và c. Xét 2 trường hợp:

1. c b a . Khi đó 5 5 3 0; 8 12 0; 8 2 0b a b b cS c a b S S b a S S c b Theo điều kiện 2 suy ra đpcm

2. a b c . Khi đó 4 18 12 12 0a c bS S S c b a

Nếu a b c b thì 0 2c a b b c c b

8 2 4 2 0

2 2 5 0b c

c b b

S S c b b bS S S b c a b

Nếu c b a b thì

0 28 2 4 2 0

2 2 5 3 3 3 0a b

a b b

b c a b b a c aS S b a a aS S S a c a b c a b

Sử dụng bổ đề với 1 suy ra đpcm.

Page 18: Chuyên đề bất đẳng thức toán học

MỘT SỐ PHƯƠNG PHÁP CHỨNG MINH BẤT ĐẲNG THỨC PHƯƠNG PHÁP DỒN BIẾN

Trang 18

B PHƯƠNG PHÁP DỒN BIẾN B.I. Giới thiệu về tư tưởng phép dồn biến

Phương pháp dồn biến là phương pháp dùng để chứng minh những bất đẳng thức trong các trường hợp đẳng thức xảy ra khi tất cả các biến số bằng nhau (cực trị đạt tại tâm), có một số biến bằng nhau (cực trị đạt được có tính đối xứng) hay khi có một biến nằm trên biên (cực trị đạt được tại biên). Ý tưởng chung là: nếu ta đưa được về trường hợp có 2 biến bằng nhau, hoặc một biến có giá trị tại biên, thì số biến sẽ giảm đi. Do đó, BĐT mới đơn giản hơn BĐT ban đầu, đặc biệt nếu BĐT mới chỉ còn 1 biến thì bằng cách khảo sát hàm số 1 biến ta sẽ chứng minh được BĐT khá đơn giản. Chính vì tư tưởng là giảm dần số biến nên phương pháp này được gọi là phương pháp dồn biến.

Hầu hết các BĐT đểu rơi vào 1 trong các dạng nói trên, nên có thể thấy ứng dụng của phương pháp dồn biến là rất lớn, đặc biệt là khi chứng minh bất đẳng thức 3 hay 4 biến. Với số biến ít như vậy, sau khi “dồn biến” việc chứng minh sẽ đơn giản hơn. Tuy nhiên, phương pháp dồn biến cũng có thể được áp dụng trong các trường hợp n biến tổng quát, bằng cách sử dụng những định lý về dồn biến tổng quát một cách linh hoạt.

B.II. Bất đẳng thức đối xứng 3 biến đạt cực trị tại tâm

Ứng dụng tư tưởng của phương pháp dồn biến vào chứng minh BĐT 3 biến đối xứng cực trị đạt tại tâm, ta rút ra cách làm sau Giả sử cần chứng minh f ( x, y, z) ≥ 0 với x, y, z thỏa mãn tính chất nào đấy. Bước 1: (Kỹ thuật dồn về 2 biến bằng nhau) Đánh giá f (x, y, z) ≥ f (t, t, z) với t là biến sao cho (t, t, z) thỏa mãn mọi tính chất của (x, y, z) Bước 2: Đánh giá f (t, t, z) ≥ 0.

Chú ý: Trong các bất đẳng thức đồng bậc, ta thường dùng kỹ thuật chuẩn hóa để lời giải ngắn gọn và đơn giản hơn. Chúng ta sẽ áp dụng tư tưởng trên vào một số ví dụ sau.

Ví dụ B.II.1: Cho x,y,z ≥ 0. Chứng minh rằng 33x y z xyz (1)

Chứng minh Đặt 3( , , ) 3 , , 0f x y z x y z xyz x y z Cách 1:

Phần

B

Page 19: Chuyên đề bất đẳng thức toán học

MỘT SỐ PHƯƠNG PHÁP CHỨNG MINH BẤT ĐẲNG THỨC PHƯƠNG PHÁP DỒN BIẾN

Trang 19

Bước 1: Chứng minh ( , , ) ( , , )f x y z f t t z với 2

x yt . Ta có: t2 ≥ xy suy ra:

3 32 23 3( , , ) ( , , ) 3 2 3 3 0f x y z f t t z x y z xyz t z t z t z xyz

Bước 2: Chứng minh ( , , ) 0f t t z

Thật vậy, 3 22( , , ) 0 2 27 0 8 0f t t z t z t z t z t z ( luôn đúng với t, z ≥ 0)

Vậy ta có điều phải chứng minh. Dấu “=” xảy ra khi a = b = c Cách 2: Bước 1: Chứng minh ( , , ) ( , , )f x y z f t t z với t xy . Ta có 2t ≤ x + y suy ra:

3 3( , , ) ( , , ) 3 2 3 2 0f x y z f t t z x y z xyz t z xyz x y t

Bước 2: Chứng minh ( , , ) 0f t t z

Thật vậy, 3 22( , , ) 0 2 27 0 8 0f t t z t z t z t z t z ( luôn đúng với t, z ≥ 0)

(đpcm) Dấu “=” xảy ra khi a = b = c

Ở đây ta nhận thấy bất đẳng thức là đồng bậc nên lời giải bài toán có thể ngắn gọn hơn bằng việc dùng kỹ thuật chuẩn hóa

Cách 3: Bước 1: Chuẩn hóa x + y + z = 1

3(1) 1 3 1 27 0xyz xyz

Bước 2: Đặt ( , , ) 1 27f x y z xyz và 2

x yt . Ta có 2t xy suy ra

2( , , ) ( , , ) 27 ( ) 0f x y z f t t z z t xy Vậy ( , , ) ( , , )f x y z f t t z Bước 3: Chứng minh ( , , ) 0f t t z Thay z = 1 – 2t ta có

22( , , ) 1 27 1 2 1 6 1 3f t t z t t t t đúng do t ≥ 0

Vậy ta có điều phải chứng minh. Dấu “=” xảy ra khi a = b = c Cách 4: Bước 1: Chuẩn hóa xyz = 1 (1) 3 3 0x y z x y z

Bước 2: Đặt ( , , ) 3f x y z x y z và t xy . Ta có 2t ≤ x + y suy ra ( , , ) ( , , ) 2 0f x y z f t t z x y t

Vậy ( , , ) ( , , )f x y z f t t z Bước 3: Chứng minh ( , , ) 0f t t z

Thay 2

1zt

ta có

2

2 2

1 2 11( , , ) 2 3 0t t

f t t z tt t

đúng do t ≥ 0

Page 20: Chuyên đề bất đẳng thức toán học

MỘT SỐ PHƯƠNG PHÁP CHỨNG MINH BẤT ĐẲNG THỨC PHƯƠNG PHÁP DỒN BIẾN

Trang 20

Vậy ta có điều phải chứng minh. Dấu “=” xảy ra khi a = b = c

Một vài nhận xét:

Trên đây là một ví dụ mở đầu cho phép dồn biến, với biến t bằng trung bình cộng hay trung bình nhân. Đây cũng là nhưng kỹ thuật chọn biến t hay dùng Bài toán chỉ nhẳm mục đích cho chúng ta cái nhìn đầu tiên về ứng dụng của tư tưởng phép dồn biến vào chứng minh bất đẳng thức.

Cách giải 3 và 4 sử dụng kỹ thuật chuẩn hóa, giúp lời giải có vẻ ngắn gọn hơn.Việc chuẩn hóa các bất đẳng thức đồng bậc cũng là một kỹ thuật hay gặp trong chứng minh bất đẳng thức. Nếu một bài toán đã chuẩn hóa(tức là BĐT có điều kiện) thì nó sẽ "gợi ý" cho chúng ta cách dồn biến (phải đảm bảo điều kiện), tuy nhiên, ngược lại một bài toán chưa chuẩn hóa (BĐTkhông điều kiện) thì chúng ta sẽ có nhiều cách để dồn biến hơn (nói chung, ta sẽ chọn cách dồn biến sao cho bảo toàn được "nhiều" biểu thức nhất trongBĐT - điều này cũng tương đương với chuẩn hóa sao cho biểu thức có dạngđơn giản nhất). Do đó, một sự phối hợp tốt giữa kĩ thuật chuẩn hóa và dồn biến là một điều cần thiết. Tuy nhiên, khi đã quen với những điều này thì các bạn sẽ thấy không có sự khác biệt đáng kể nào giữa chúng.

Sử dụng phương pháp dồn biến trong chứng mính bất đẳng thức Cauchy ở trên rất đơn giản do không khó để chứng minh f (x , y , z) ≥ f ( t , t , z ) ≥ 0. Tuy nhiên, thực ra các bài toán thường không bao giờ đơn giản như vậy. Chúng ta sẽ xét tiếp những ví dụ khác để thấy hiệu quả của phương pháp dồn biến.

Ví dụ B.II.2: [MOSP 2001] Cho , , 0a b c và 1abc .CMR:

4 1a b b c c a a b c

Chứng minh Đặt

( , , ) 4 1f a b c a b b c c a a b c

2 2 22 4 1abc a b c a b c bc b c a b c

Không mất tính tổng quát ta giả sử max , ,a a b c

Có 2 22( , , ) ( , , ) 4d f a b c f a bc bc b c a a b c bc

Xét:

2 22 2 2

2 2

1 1 1 1 1 24 4 4

1 4 14 0

A a a b c bc a b c abc a a b c bc

a aa abc

(Do ax , ,a m a b c và 1abc .nên 41 4 0bcbc

)

Vậy suy ra 0d hay ( , , ) ( , , )f a b c f a t t (với 0a t và 2 1at )

Có 3 2 2 33 2

2 4( , , ) 2 2 4 4 8 4 2 8 8f a t t t a t at a t t tt t

(do 2

1at

)

Page 21: Chuyên đề bất đẳng thức toán học

MỘT SỐ PHƯƠNG PHÁP CHỨNG MINH BẤT ĐẲNG THỨC PHƯƠNG PHÁP DỒN BIẾN

Trang 21

23 21 1 12(1 ) 4 1 0t t tt t t

(do 1t )

Vậy ( , , ) ( , , ) 0f a b c f a t t Đpcm Dấu “=” xảy ra khi a=b=c=1

Ở đây, chúng ta không thể chứng minh trực tiếp ( , , ) ( , , )f a b c f a t t mà phải đánh giá thông qua việc coi a = max{a,b,c} Việc sắp xếp thứ tự của các biến cũng là một kỹ thuật thường gặp trong chứng minh bất đẳng thức. Nhắc lại là nếu BĐT 3 biến đối xứng thì ta có thể giả sử a ≥ b ≥ c (hoac a ≤ b ≤ c), còn trong trường hợp BĐT 3 biến hoán vị vòng quanh thì ta có thể giả sử a = min{a, b, c} (hoặc a = max{a, b, c}).

Phương pháp dồn biến thường cũng được dùng nhiều trong các bất đẳng thức có điều kiện. Với các BĐT có điều kiện, ta sẽ ko dồn biến như thường mà phải làm sao để điều kiện luôn được thoả mãn. Chẳng hạn với ĐK 3xy yz zt , khi chúng ta muốn dồn 2 biến chẳng hạn y và z bằng nhau, tức là ( , , ) ( , , )f x y z f x t t thì biến t ở đây không phải là 1 đại lượng trung bình của y, z mà là đại lưọng thoả mãn 22 1xt t . Ta xét một vài ví dụ sau.

Ví dụ B.II.3: (VMO) Cho x, y, z là các số thực thoả mãn 2 2 2 9x y z Chứng minh rằng: 2( ) 10x y z xyz

Chứng minh

Đặt f(x,y,z) = 2( )x y z xyz và t = 2 2

2y z

Ko mất tính tổng quát ta giả sử x y z Xét d = f(x, y, z) – f(x, t, t) = 22( 2 ) ( )y z t x yz t

Dễ thấy 2 0y z t và 2 0yz t nên khi 0x thì 0d Ta xét 2 trường hợp: Nếu x 0, khi đó f(x, y, z) f(x, t, t). Ta sẽ CM f(x, t, t) 10

Thật vậy, thay t = 2 2

2y z =

292

x

f(x, t, t) = 2 212 2 2(9 ) (9 )2

x x x x = g(x)

Do 0x và 2 9x nên 3,0x

Vậy ta xét hàm số g(x) = 2 212 2 2(9 ) (9 )2

x x x x / 3,0

g’(x) =2

2

3 5 42 2 18 2x x

x

Có g’(x) = 0 với 3,0x 1x

Ta có BBT

Page 22: Chuyên đề bất đẳng thức toán học

MỘT SỐ PHƯƠNG PHÁP CHỨNG MINH BẤT ĐẲNG THỨC PHƯƠNG PHÁP DỒN BIẾN

Trang 22

x -3 -1 0 g’(x) + 0 –

g(x) -6

10

6

2 Từ BBT suy ra : g(x) 10 f(x, y, z) f(x, t, t) = g(x) 10 đpcm Dấu “=” xảy ra khi x = -1, y = z =2. Nếu x > 0 y > 0, z > 0. Khi đó ta sẽ xử lí bài toán bằng cách xét 2 trường hợp

- Nếu x 34 thì f(x, y, z)= 3

2 2 2 3 272( ) 2 3( ) 2 27 104 64

x y z xyz x y z

- Nếu x < 34 thì f(x, y, z)= 2 2 3 32( ) 2 2( 2 18 104 4

x y z xyz y z

Vậy BĐT được cm, dấu “=” xảy ra khi chẳng hạn 1x , 2y z

Ví dụ B.II.4: [Mathlinks] Cho a, b, c là các số dương có tích bằng 1. CMR 1 1 1 6 5a b c a b c

Chứng minh

Đặt f(a,b,c) = 1 1 1 6 5a b c a b c

Không mất tính tổng quát ta có thể giả sử max , ,a a b c

Xét d = f(a,b,c) – f(a, bc , bc )

=

2 1 6( )2

b cbc a b c a bc

Do max , ,a a b c nên a bc ; 2b c bc

1 6 1 2 1( )( 2 )) 9 1 03 3( )( 2 )

a b c a bc bcbc bc bca b c a bc

(1)

Từ (1) suy ra d 0. Vậy f(a,b,c) f(a,t,t) (với 0a t và 2 1at ) (*)

Xét f(a,t,t) = 2

23

1 2 6 2 65 52 1 2

tta t a t t t

(do 2

1at

)

24 3 2

3

12 4 4 2

2 1t

t t t tt t

(2)

Đặt g(t) = 4 3 22 4 4 2t t t t / (0,1] ( do 0a t và 2 1at nên 0 1t )

Có g’(t) = 3 28 12 8 1t t t '( ) 0 ( )( )( ) 0g t t x t y t z (với , 0,0 1y z x )

Page 23: Chuyên đề bất đẳng thức toán học

MỘT SỐ PHƯƠNG PHÁP CHỨNG MINH BẤT ĐẲNG THỨC PHƯƠNG PHÁP DỒN BIẾN

Trang 23

t = x Ta có BBT t 0 x 1 g’(t) + 0 -

g(t) g(x) 2 3

Từ BBT 0

( ) lim ( ) 2 ( ) 0 / (0;1]t

g t g t g t

2

3

10

2 1t

t t

( do t > 0) nên từ (1) ( , , ) 0f a t t (**)

Từ (*) & (**) ( , , ) 0f a b c đpcm (Dấu “=” xảy ra khi a = b = c = 1)

Ví dụ B.II.5: Cho x, y, z ≥ 0 thỏa mãn 2 2 2 3x y z . Chứng minh rằng:

7 12 9xy yz zx xyz

Chứng minh Đặt ( , , ) 12 9 7 12 9 7 7f x y z xyz xy yz zx xyz xy z x y

2 22 2 2 2 2 2

2 2

2 22 2

22

2 2

2 2

32

( , , ) ( , , )

9 7 7 2 0

9 7 7 22

149 7

2( )

(9 7) 2 14

x yt t t z x y z

f x y z f t t z

z xy t xy t z x y t

x yxy z z x y x y

z x yx y z

x y x y

z x y x y z

Do 2 22x y x y . Ta sẽ chứng minh

2 2 279 7 2 7 9 2 3 7z x y z zz

(*)

Đặt z = min{x,y,z} ta có 0 ≤ z ≤ 1 79 2z

và 22 3 6z (*) đúng.

Vậy f(x, y, z) ≥ f( t, t, z). Ta sẽ chứng minh f(t, t, z) ≥ 0 23 2z t ta có

2 2 2 2( , , ) 9 3 2 7 14 3 2 12f t t z t t t t t với 31,2

t

Đặt r = 1/t2 ta phải chứng minh

Page 24: Chuyên đề bất đẳng thức toán học

MỘT SỐ PHƯƠNG PHÁP CHỨNG MINH BẤT ĐẲNG THỨC PHƯƠNG PHÁP DỒN BIẾN

Trang 24

2( ) 9 3 7 14 3 2 12 0g r r rr

9 21 3 3'( ) 12 7 123 2 3 2 3 2

g rr r r r r r r

với 2 ,1

3r

'( ) 0 1g r r với 2 ,13

r mà 2 2' 0 '( ) (1) 0 ,1

3 3g g r g r

Vậy ( ) (1) 0g r g . Ta có điều phải chứng minh. Dấu = xảy ra khi x = y = z = 1

Qua các ví dụ trên, ta thấy việc khảo sát hàm số là công việc rất hay gặp trong phép chứng minh thứ 2 của phương pháp dồn biến. Việc nắm vững những kiến thức về đạo hàm, hàm số vì vậy là rất quan trọng.

Dồn biến về giá trị trung bình rất hữu dụng. Tuy nhiên các cách dồn biến là vô cùng phong phú và uyển chuyển. Như đã nói ở trên, việc dùng kỹ thuật chuẩn hóa trong phương pháp dồn biến cũng rất hay gặp. Chúng ta sẽ xét một ví dụ minh họa cho điều này

Ví dụ B.II.6: (Iran 1996) Chứng minh rằng với a, b, c ≥ 0 th ì

2 2 2

1 1 1 94

ab bc caa b b c c a

Chứng minh Ở đây, dẫu “=” xảy ra ngoài a = b = c còn có , 0a b c Vì BĐT là đồng bậc nên ta sẽ chuẩn hóa 1ab bc ca (*).

Đặt 2 2 2

1 1 1( , , )f a b ca b b c c a

. Ta cần chứng minh 9( , , )4

f a b c

Với tư tưởng của dồn biến, ta nghĩ đến việc chứng minh ( , , ) ( , , )f a b c f t t c . Do ( t, t, c)

cũng thỏa mãn (*) ta có 2 2 1t tc .

2 2 2 22

1 1 1 1 2( , , ) ( , , )4

d f a b c f t t cta b b c c a t c

Ta có 22 2t ct ab bc ca c t a c b c . Từ đó:

22

2

2 2 22

2 2

2 2 22 2

2 2 2

2 2 1 1 24

2

4

a b t a c b c c t a c b c a c b c

a ba c b c

a c b c

t a b t a bd

a b b cb c t a c b c

a b t a b a ba c b ca c b c b c t

Page 25: Chuyên đề bất đẳng thức toán học

MỘT SỐ PHƯƠNG PHÁP CHỨNG MINH BẤT ĐẲNG THỨC PHƯƠNG PHÁP DỒN BIẾN

Trang 25

22 2 2 2 2

1 2

4

t a ba ba c b c a c b c b c t

Giả sử min , ,c a b c thì 33

c và 33

t

2

2 , 2t a b a c b c a c b c a c b c c t a b t

Do đó d ≥ 0, ta đã chứng minh được ( , , ) ( , , )f a b c f t t c

Thay 21

2tct

ta có

22 42 4 2

2 2 2 22 22 2 2 2 2 2

22 4 6 6 4 2 2 2

2 22 2 2 2

1 321 2 1 8 33 2 1( , , )4 41 1 4 1 4 1

2

1 7 15 9 9 18 9 1 1 3 9 94 44 1 4 1

t tt t tf t t ct tt t t t t tt

t

t t t t t t t t

t t t t

Bài toán được chứng minh. Dấu bằng xảy ra khi a = b = c hoặc a = b , c = 0 và các hoán vị.

Đây là một bài toán rất khó. Ngay cả khi dùng phương pháp dồn biến, việc chứng minh

( , , ) ( , , )f a b c f t t c và 9( , , )4

f t t c đều khá phức tạp, đòi hỏi kỹ thuật biến đổi phải rất

thành thạo. Đây cũng là một ví dụ điển hình thể hiện việc chọn biến t thích hợp với điều kiện của đề bài, cũng như kết hợp chuẩn hóa và dồn biến để chứng minh bất đẳng thức.

Ưu điểm của cách dồn biến cũng thể hiện rất rõ trong ví dụ này. Với bất đẳng thức 3 biến đối xứng cực trị đạt tại tâm, có 3 hay 2 biến bằng nhau, việc sử dụng dồn biến tỏ ra thật hiệu quả.

B.III. Dồn biến bằng kỹ thuật hàm số Đây là một kĩ thuật rất quan trọng của phương pháp dồn biến. Tuy nhiên chúng tôi giới

thiệu nó ngay sau phần cơ bản nhất là nhằm trang bị cho các bạn một kĩ thuật cần thiết trước khi đi qua các mục sau. Hơn nữa, chúng tôi nghĩ rằng khi đã quen với nó thì các bạn sẽ không còn phải phân biệt cực trị đạt tại tâm hay tại biên, và do đó mục tiếp theo sẽ nhẹ nhàng hơn.

Trong $2 chúng ta thấy rằng để chứng tỏ f(x, y, z) ≥ f(t, t, z) ta chỉ việc xét hiệu d = f(x, y, z) − f(t, t, z) rồi tìm cách đánh giá sao cho d ≥ 0. Tuy nhiên, đó là vì dạng BĐT quá đơn giản, phù hợp với các biến đổi đại số. Giả sử ta phải làm việc với biểu thức f có dạng, chẳng hạn, như: f(x, y, z) = xk + yk + zk với k > 0 thì các cách biến đổi đại số sẽ trở nên rất cồng kềnh và phức tạp.

Kĩ thuật hàm số dùng để giải quyết các trường hợp như vậy. Ý tưởng chính thế này, chẳng hạn để chứng minh f(x, y, z) ≥ f(x, t, t) với t = (y+z)/2, ta xét hàm: g(s) = f(x, t+s, t−s) với s ≥ 0. Sau đó chứng minh g tăng với s ≥ 0 (thông thường dùng công cụ đạo hàm rất tiện lợi), suy ra g(s) ≥ g(0), ∀ s ≥ 0, và ta sẽ thu được điều mong muốn. Một trong những ví dụ quen thuộc với các bạn là dồn biến bằng hàm lồi, tuy nhiên dưới đây chúng ta sẽ quan sát kĩ thuật dồn biến trong bối cảnh tổng quát hơn, còn vấn đề về hàm lồi sẽ được trở lại ở một mục sau trong bài toán với n biến.

Page 26: Chuyên đề bất đẳng thức toán học

MỘT SỐ PHƯƠNG PHÁP CHỨNG MINH BẤT ĐẲNG THỨC PHƯƠNG PHÁP DỒN BIẾN

Trang 26

Chúng tôi nhấn mạnh rằng, đây là một kĩ thuật khó, bởi nó chứa đựng những nét rất tinh tế của phương pháp dồn biến. Những ví dụ sau đây thể hiện rất rõ vẻ đẹp và sức mạnh của phương pháp dồn biến.

Ví dụ B.III.1: Cho 0k và , , 0a b c .CMR:

3min 2,2

k k k

ka b c

b c c a a b

(*)

Chứng minh

Bước 1: Ta chỉ cần chứng minh bất đẳng thức khi 322k

ln 3 1ln 2

k (!)

Thật vậy

Nếu ln 3 ln 3 3 30 1 1 2 2 min 2,ln 2 ln 2 2 2k kk k

Do ln 30 1ln 2

k

nênln3 ln3 ln31 1 1ln 2 ln 2 ln 2

k k ka b c a b cb c c a a b b c c a a b

Vậy cần CM ln3 ln3 ln31 1 1ln 2 ln 2 ln 2

2a b cb c c a a b

hay nhận xét (!) đúng.

Nếu ln 3 1ln 2

k làm tương tự .

Bước 2: Với ln 3 1ln 2

k ,ta sẽ cm BĐT đúng.

Không mất tính tổng quát có thể giả sử 1a b c và b c a .

Đặt 2

b ct và

2b cm

suy ra , , 1 2b t m c t m a t . Khi đó:

1 2(*) ( ) 22 1 1

k k kt t m t mf mt t m t m

với ln 3 1

ln 2k

Vì c a nên 3 1 0t m , và 1 2b c t nên 1 12 3

t

Ta sẽ khảo sát ( )f m trên miền 0,3 1m t với 1 1,3 2

t là hằng số.

Ta có 1 1 1 1

1 1 1 1

( ) ( ) ( ) ( )'( ) 0(1 ) (1 ) (1 ) (1 )

k k k k

k k k kk t m k t m k t m k t mf m

t m m t t m m t

1( ) ln( ) ln( ) ln(1 ) ln(1 ) 01

kg m t m t m t m t mk

Ta có 1 1 1 1 1'( ) 01 1 1

kg mt m t m k t m t m

(1) 2 2 2 2

2 1 2(1 ) 1 10 0( )( ) 1 (1 )(1 ) 1 (1 )

t k t t k tt m t m k t m t m t m k t m

Page 27: Chuyên đề bất đẳng thức toán học

MỘT SỐ PHƯƠNG PHÁP CHỨNG MINH BẤT ĐẲNG THỨC PHƯƠNG PHÁP DỒN BIẾN

Trang 27

Do ln 3 1ln 2

k nên 1 21

kk

suy ra để chứng minh (1) ta cần phải chứng minh

2 3 2 22 2 2 2

2(1 ) 0 ( ) 4 3 3 2 0(1 )

t t u m t t t tm mt m t m

Ta có 21 1'( ) 2(3 2) 0 , ( ) (3 1) 2(3 1)(2 1) 03 2

u m t m t u m u t t t

(1) đúng '( ) 0 ( )g m g m đồng biến ( ) (0) 0 '( ) 0g m g f m ( )f m đồng biến ( ) (0)f m f

Bước 3: Ta cần chứng minh 1 2 1(0) ( ) 2 2, 0,2 1 3

k kt tf h t tt t

1 1

11 21 1

2 (1 2 )'( ) 0 2 (1 )(1 2 )(1 ) 2

k kkk k

k k kkt k th t t t t

t t

(2)

Trong bất đẳng thức cuối, vế trái là hàm đồng biến theo t và vế phải là hàm nghịch biến theo

t, và lưu ý la 13

t nên để chứng minh (2) ta cần chứng minh:

121 1 1 22 1 1

3 3 3

kkk

. Bất đẳng thức này đúng nên ( )h t nghịch biến, suy ra

1( ) 2.3

h t h

Vậy bài toán được chứng minh.

Nhận xét: Để thấy được nét đẹp của bài toán này, ta xét các trường hợp đặc biệt.

Trường hợp 1k , ta thu được BĐT Nesbit: 32cyc

ab c

Trường hợp 12

k , ta có BĐt thức sau:

2cyc

ab c

Đây là 1 bài toán khá thú vị với 1 lời giải đơn giản sử dụng BĐT thức AM - GM: 2 2( ) 2

2 ( )cyc cyc

a a a b cb c a b ca b c

Ví dụ B.III.2: (Phạm Kim Hùng) Cho các số thực dương a, b, c có tích bằng 1. Chứng minh rằng:

a) 42 2 281 1 1 1 8a b c a b c

b) 63 3 364 1 1 1a b c a b c

Chứng minh

a) Đặt 4 2 2 2( , , ) 8 81 1 1 1f a b c a b c a b c . Giả sử a ≥ b ≥ c ta có c ≤ 1.

Xét hàm số

Page 28: Chuyên đề bất đẳng thức toán học

MỘT SỐ PHƯƠNG PHÁP CHỨNG MINH BẤT ĐẲNG THỨC PHƯƠNG PHÁP DỒN BIẾN

Trang 28

4 22 2 2

2

4 22 2 2 2 2

2

( ) ( , , ) 8 81 1 1 1

8 81 1 1

b b bg t f ta c ta c c a tt t t

b bta c c a b a tt t

với ,1bta

ta có:

3 22 2

2 3

32

2 2

2'( ) 32 81 2 1

32 162 1

b b bg t a ta c a t ct t t

b b b ba ta c a ta ct t t t

Đặt bd tat

, dễ thấy c ≤ 1 nên 12dc

c . Vậy

3 2 2332 32.9 32.9. 432 216 1 162 12

d c d c d d d c d c do c ≤ 1

Do 2,1 0b bt aa t

. Vậy '( ) 0g t g(t) đồng biến trên ,1ba

(1) ( , , ) ( , , )bg g f a b c f s s ca

với s ab

Thay 1sc

ta có

4 2

2

2 9 9 5 35 4 3 22 2 2 2

2 1( , , ) 8 81 1 1

1 8 16 24 96 87 78 99 120 21 94 47 0

f s s c c ccc

c c c c c c c c c c cc

Bất đẳng thức đã được chứng minh. Dấu = xảy ra khi a = b = c = 1

b) Đặt 6 3 3 3( , , ) 64 1 1 1f a b c a b c a b c . Giả sử a ≥ b ≥ c nên c ≤ 1.

Xét hàm số ( ) ( , , )bg t f at ct

với ,1bta

6 63 3

3 3 3 3 3 3 3 33 3( ) 64 1 1 1 64 1 1b b b bg t at c a t c at c a t a b c

t t t t

5 3 23 2 3

2 6

5 22 2 3

2 2 2

3'( ) 6 64 3 1

6 192 1

b b b tg t a ta c a t ct t t

b b b ba ta c a a t ab ct t t t

Page 29: Chuyên đề bất đẳng thức toán học

MỘT SỐ PHƯƠNG PHÁP CHỨNG MINH BẤT ĐẲNG THỨC PHƯƠNG PHÁP DỒN BIẾN

Trang 29

Vì ,1bta

nên g’(t) ≥ 0 nếu 5 2

2 2 3232 1b bta c t a ab c

t t

Đặt 2

2 22

bd t a abt

, ta có: 5 4

3b bta c ta ct t

2

2 2 23 2 2 2 3 6 72 36 1 32 1bcd ab tac d d d c d ct

vì c ≤ 1

Vậy g’(t) ≥ 0 hay g(t) đồng biến trên ,1ba

, do đó (1) bg ga

tức là

( , , ) ( , , )f a b c f s s c với s ab .

Thay 1sc

, ta được:

6 2

3

6 4 3

2 1( , , ) 64 1 1

6412 4 32 112 0

f s s c c cc c c

c c c c c cc c

(do c ≤ 1). Ta có điều phải chứng minh

Đây là một ví dụ về cách dồn biến về trung bình nhân. Có thể thấy việc biến đổi, đánh giá cũng như khảo sát hàm số là khá phức tạp. Để nghĩ ra những lời giải như vậy cần có một sự kiên trì rèn luyện. Vì vậy điều quan trọng nhất của chúng ta là phải có ý thức làm đến nơi đến chốn, không được bỏ dở giữa chừng, dù phải đối mặt với những biến đổi rất khủng khiếp. Khi giải được bài toán chúng ta sẽ cảm thấy mình đã nhận được thành quả xứng đáng.

Tiếp theo là một bất đẳng thức mà đề khá đơn giản, nhưng việc giải nó thì không dễ chút nào. Phương pháp dồn biến là một phương pháp hiệu quả để giải bài toán này.

Ví dụ B.III.3: Cho , , 0

3a b ca b c

. Tìm giá trị lớn nhất của 2 2 23 3 3ab bc caS

c a b

Chứng minh Không mất tổng quát a ≥ b ≥ c.

Đặt a = s + t, b = s – t thì

2 2

2 2 2

( ) ( )( )33 3

c s t c s t s tS f tcs t s t

Khảo sát f(t) trên 0,t s c

2 2 2 2

2 2 2 2 22 2

2 22 2

2 2 2

2 2 2'( )33 33 3

84 2 , 0, , 3 , 33

c s t c s tc c tf tcs t s ts t s t

cst s t u vcst t t s c u s t v s tuv u v c

Page 30: Chuyên đề bất đẳng thức toán học

MỘT SỐ PHƯƠNG PHÁP CHỨNG MINH BẤT ĐẲNG THỨC PHƯƠNG PHÁP DỒN BIẾN

Trang 30

Có 2 2s a b , vậy 31,2

s nên (3 2 ) 1cs s s . Mặt khác 23 4,u s t

2 23 3v s t c do 2

40, (*)3

cst tt s cuv c

Theo bất đẳng thức Cauchy:

2 22 22 2 2 2 23 3 16u v s t s t s t (1)

Và 32 2 2

2 2 2 2 4 3 38 3 16 3 3 43

cs s t ccs u v c cs s t c

Thay c = 3 – 2s và t ≤ s – c = 3s – 3 ta có:

2 22 2 2 2

2 2 2 2 2

4 3 3 4 3 2 6 3 3 3 2

12 8 6 9 18 9 9 12 4 36 30 6 6 2 3 12

cs s t c s s s s s

s s s s s s s s s s

Do 31,2

s . Từ đó suy ra 2 48 3 4cs u v c (2)

Từ (1) và (2) ta có 2 2

2 2 2

8(**)

3cst s t u v t

u v c

Từ (*) và (**) suy ra f ‘(t) ≤ 0 với 0,t s c

Vậy

2 2

22 2 2

2 2 (3 2 )( ) (0) ( )3 3 3 3 3 2

cs s s s sf t f g ss c s s

(1)

Khảo sát g(s) trên 31,2

s . Ta có

2 22 2

2 222

22 22 2

2 2 222 22 2

2 3 3 2 8 126 8 3 2 6 4'( )

3 3 3 2

108 3 4 1 3 66 24 18 12 24

3 3 3 2 3 3 3 2

s s s ss s s s sg s

s s

s s s s ss s s s

s s s s

Dễ thấy s2 – 3s + 4 > 0 và – s2 -3s + 6 = 33 3 33 32 3

s s

nên g ‘(s) dương trên

(1,s0) và âm trên (s0,3/2) với 033 3

2s

Vậy 31,2

s thì 0

11 33 45( )24

g s g s (2)

Vậy giá trị lớn nhất của S là 11 33 4524 . Giá trị đạt được khi xảy ra đẳng thức ở (1) và (2)

tức là t = 0 và s = s0, tức là a = b = s0 và c = 3 – 2s0 hay a = b = 33 32 , c = 6 33 và các

hoán vị

Page 31: Chuyên đề bất đẳng thức toán học

MỘT SỐ PHƯƠNG PHÁP CHỨNG MINH BẤT ĐẲNG THỨC PHƯƠNG PHÁP DỒN BIẾN

Trang 31

B.IV. Bất đẳng thức 3 biến cực trị đạt tại biên Nếu như trong phần trước chúng ta có thể hiểu "dồn biến" là "đẩy hai biến lại gần nhau", thì trong trường hợp này ta phải hiểu "dồn biến" nghĩa là "đẩy 1 biến ra biên". Chẳng hạn như xét BĐT f(x, y, z) ≥ 0 với x, y, z ≥ 0, ta có thể hi vọng vào đánh giá f(x, y, z) ≥ f(0, s, t), trong đó s, t là các đại lượng thích hợp sinh ra từ các biến a, b, c (ta sẽ gọi đây là kĩ thuật dồn 1 biến ra biên). Tất nhiên ta sẽ chọn s, t sao cho hiệu d = f(x, y, z) – f(0, s, t) là đơn giản và có thể đánh giá thuận lợi. Cuối cùng ta chỉ việc kiểm chứng f(0, s, t) ≥ 0. Từ lý thuyết như vậy đến thực hành cũng là cả một vấn đề. Chúng ta sẽ xét một số ví dụ để rút ra những kinh nghiệm cho mình về việc dồn biến ra biên

Ví dụ B.IV.1: Cho a, b, c ≥ 0, ab + bc + ac = 1 (*). Chứng minh rằng 1 1 1 12

2a b c b a c

Chứng minh

Dễ thấy đẳng thức không xảy ra tại tâm (a = b = c = 13

). Ta nghĩ đến việc một biến bẳng 0.

Ta sẽ chứng minh ( , , ) (0, , )f a b c f s t với s, t thích hợp. Lấy 1,s a b ta b

thỏa mãn (*)

Việc xét hiệu ở đây khá khó khăn. Ta biểu diễn ( , , )f a b c về các bến x = a + b và c như sau:

2

2

1 1 1 1( , , )

21 1 2 2 1 ( )1

b c c af a b ca b c b a c a b b c c a

c a b c b c c a c x c g ca b xb c c a c

Ta có:

22 2

22

2

2 2 22 2

3 32 2 2 2

2211 2 2 1 .

12 2 2 1'( )1

1 1 2 2 1 1 1

1 2 2 1 1 2 2 1

cccc c x c

cc x cg cc

c c c c x c c cx c c

c c x c c c x c

Giả sử c = max {a, b, c} ta có 2 21 0c cx ab c . Vậy g ‘ (c) ≤ 0. Do đó g (c) nghịch

biến. Lại có cx = ca + ab ≤ ab + bc + ca ≤ 1 hay 1cx

.

Do đó

Page 32: Chuyên đề bất đẳng thức toán học

MỘT SỐ PHƯƠNG PHÁP CHỨNG MINH BẤT ĐẲNG THỨC PHƯƠNG PHÁP DỒN BIẾN

Trang 32

2 2

23 2

2 22

2

2

2 12 1 11 1 1 2 2 1 1( ) .( 1) 11

1 ( , , ) (0, , )1

xx xx x x xx xg c g xx x xx x xx

xxx f a b c f s t

xx

Ta còn phải chứng minh 2

1 1( ) 21 2

xh x xxx

2

2 22

2 3 32 2

11 1 1 1 2 1 12 1'( ) 1

12 2 2 22 1 2 1

x xxx x x xx xh x x

xx x x x x x xx x x x

3 32 22 2 2 2

3 3 3 22 2 2 2

3 4 3 2

3 3 22 2 2

1 2 1 1 2 11 1

2 1 2 1 1 2 1

1 2 4 2 1

2 1 1 2 1

x x x x x xx x

x x x x x x x x x

x x x x x

x x x x x x

Do x ≥ 0 nên h’(x) dương khi x > 1 và âm khi x < 1. Từ đó ta thấy h(x) ≥ h(1) = 122

với

x ≥ 0. Bất đẳng thức đã được chứng minh. Dấu bằng xảy ra khi c = 0 a = b = 1 và các hoán vị.

Việc dồn biến ra biên ở đây rất tinh tế, thể hiện ở cả việc chọn s, t và việc biến đổi f (a, b, c) thành g(c) như đã làm. Chúng ta có thể thấy việc quy từ bất đẳng thức 3 biến về 2 biến và rồi 1 biến trong chứng minh trên. Trong các bước chứng minh, việc dùng đạo hàm là hết sức quan trọng.

Nói riêng về việc biến đổi bất đẳng thức từ 3 biến thành 2 biến, đây là một kỹ thuật khá khỏ và tinh tế, đòi hỏi một sự thành thạo về kỹ năng biến đổi. Tuy nhiên, nếu thành thạo thì đây cũng là một công cụ hết sức hữu dụng để chứng minh bất đẳng thức bằng phương pháp biến đổi. Việc biến đổi nhiều khi cũng thật phức tạp. Hy vọng chúng ta sẽ có đủ kiên trì để rèn luyện kỹ thuật này.

Ví dụ B.IV.2: (Jack Grafukel) Cho , , 0.a b c Chứng minh rằng: 54

a b c a b ca b b c c a

(1)

Nhận xét Ta xét xem dấu = xảy ra khi nào. Do a b c không thoả mãn nên một cách tự nhiên ta nghĩ đến trường hợp biên là 0c , khi

đó bất đẳng thức (1) trở thành 54

a b a ba b

(2)

Page 33: Chuyên đề bất đẳng thức toán học

MỘT SỐ PHƯƠNG PHÁP CHỨNG MINH BẤT ĐẲNG THỨC PHƯƠNG PHÁP DỒN BIẾN

Trang 33

Chuẩn hoá 1a b . Ta có (1)25 11 0

4 2b b b

( luôn đúng)

Vậ bất đẳng thức xảy ra khi 3 0, 0a b c (và các hoán vị). Do dấu bằng xảy ra khi cả 3 biến rời nhau nên khôn thể thực hiện phương pháp dồn biến về 2 biến bằng nhau. Ta sẽ thực hiện theo phương pháp dồn một biến về biên.

Chứng minh Không mất tính tổng quát ta có thể giả sử ax , ,a m a b c , chuẩn hóa cho 1a b c .

Đặt 2

a ct và

2a cs

, thì , , 1 2a t s c t s b t

Khi đó bất đẳng thức (1) 1 2 541 1 2

t s t t sf ss t t s t

(3)

Ta sẽ chứng minh ax 0 , 1f s m f f với 0;s t

Ta có

3 32 2

1 1 2 1'1 22 1 2 1

t s tf ss t ts t t s

Vì chưa xác định được dấu của s bên ta đạo hàm tiếp

3 5 52 2 2

3 3 1 21''1 4 1 4 1

t s tf s

s t s t t s

.Sử dụng 1 2 0b t suy ra

5 7 7 7 72 2 2 2 2

15 15 1 2 15 1 29 18 3 33''' 04 1 8 1 8 1 8 1 8 1

t s t ts tf ss t s t t s s t t s

Do ''' 0 0;f s s t nên theo định lí Rolle thì phương trình ' 0f s có tối đa 2 nghiệm

trên 0; t . Mặt khác dễ dàng chứng minh ' 0 0f và ' 0f t , nên 'f s chỉ có thể đổi

dấu tôi đa 1 lần trên 0; t , và hơn nữa 'f s chỉ có thể có một trong các dạng sau:

' 0, 0;f s s t hoặc ' 0, 0;f s s t Hoặc 'f s có dạng 0 trên 0; t . Tuy

nhiên trong cả 3 trường hợp thì f s cũng chỉ có thể đạt cực trị tại biên. Từ đó suy ra

ax 0 , 1f s m f f với 0;s t

Dễ dàng chứng minh 504

f và 54

f t nên 5ax 0 , 14

f s m f f

Chú ý: Bằng cách tương tự ta có thể chứng minh bài toán tổng quát sau:

“Cho , , 0a b c và 0;1k Khi đó ta có BĐT:

1 k

kk k ka b c C a b c

a b b c c a

Với 21kC k k ”

Trong trường hợp 3 biến, thông thường chúng ta cố định 1 biến và thay thế 2 biến còn lại. Tuy nhiên, đôi khi chúng ta có thể làm khác hơn bằng cách chỉ thay thế 1 biến hoặc thay thế cả 3 biến. Sau đây là một ví dụ về phương pháp dồn biến ra biên trong bất đẳng thức

Page 34: Chuyên đề bất đẳng thức toán học

MỘT SỐ PHƯƠNG PHÁP CHỨNG MINH BẤT ĐẲNG THỨC PHƯƠNG PHÁP DỒN BIẾN

Trang 34

hoán vị, cũng là ví dụ minh họa việc cố định 2 biến và thay thế 1 biến. Lời giải của bài toán khá đơn giản, giúp ta hiểu rõ hơn về tư tưởng cơ bản của dồn biến ra biên.

Ví dụ B.IV.3: Cho a, b, c ≥ 0, chứng minh rằng

22 2 2 4a b c a b c a b b c c a

Chứng minh

Đặt 22 2 2( , , ) 4f a b c a b c a b c a b b c c a Không mất tổng quát, giả

sử c = min{a, b, c}. Nếu a ≥ b ≥ c thì VT ≤ 0 BĐT hiển nhiên đúng. Vì vậy ta chỉ cần xét b ≥ a ≥ c.

2 22 2 2 2 2

22 2 2 2 2 2 2

22 2 2 2 2 2 2 2

22 2 2 2 2 2

( , , ) ( , ,0) 4

( ) 4

( ) 4

( ) 4

f a b c f a b a b c a b a b a b c b c c a a b ab

a b c a b b a a b c bc ab c ac ab a b

a b c a b b a a b bc c ac c bc ab c ac

a b c a b b a c a b a b c bc a

2

22 2 2 2 2 2 2 2 2( ) 4

0

b c ac

a b c a b b a c a b ab c

Mà 2 22 2 2 2 2 2( , ,0) 4 2 0f a b a b b a ab a ab b

Vậy ta đã chứng minh xong. Dấu bằng xảy ra khi 2 20, 2 0c a ab b hay

0, 3 1c a b và các hoán vị.

B.V. Định lý dồn biến mạnh S.M.V và bất đẳng thức 4 biến

Định lý S.M.V BỔ ĐỀ Giả sử a1, a2, …, an là dãy sổ thực tùy ý. Ta thực hiện liên tiếp phép biến đổi

sau 1. Chọn i, j 1, 2,..., n là 2 chỉ số sao cho

ai = min {a1, a2, …, an} aj = max {a1, a2, …, an}

2. Thay ai , aj bởi 2

i ja a (nhưng vẫn giữ đúng thứ tự của chúng trong dãy số)

Khi đó sau vô hạn lần thực hiện biến đổi nói trên thì mỗi số ai đều tiến tới giới hạn 1 2 ... na a aa

n

CHỨNG MINH Phép biến đổi nói trên, từ giờ trở đi ta sẽ gọi là phép biến đổi ∆. Ký hiệu dãy ban đầu là

1 1 11 2, ,..., na a a . Sau một phép biến đổi ta được dãy mới ký hiệu là 2 2 2

1 2, ,..., na a a . Làm tương tự,

từ dãy 1 2, ,...,k k kna a a ta thu được dãy mới kí hiệu là 1 1 1

1 2, ,...,k k kna a a . Khi đó với mọi số

nguyên 1,i n ta phải chứng minh

Page 35: Chuyên đề bất đẳng thức toán học

MỘT SỐ PHƯƠNG PHÁP CHỨNG MINH BẤT ĐẲNG THỨC PHƯƠNG PHÁP DỒN BIẾN

Trang 35

1 2 ...lim ,k nik

a a aa a an

Ta đặt 1 2min , ,...,k k kk nm a a a và 1 2max , ,...,k k k

k nM a a a

Dễ thấy phép biến đổi ∆ không làm tăng giá trị của Mk và không làm giảm giá trị của mk. Vì mk và Mk đều là các dãy bị chặn nên tồn tại

lim kkm m

lim kk

M M

Ta phải chứng minh m = M. Phản chứng giả sử M > m, Đặt dk = Mk – mk. Ta có một nhận xét đơn giản sau đây Nhận xét. Giả sử sau một số phép biến đổi ∆ dãy 1 1 1

1 2, ,..., na a a trở thành dãy 1 2, ,...,k k kna a a

sao cho 1 1

2kM mm

thì ta có 1 12 2

M mm

Thật vậy, không mất tính tổng quát của bài toán ta giả sử 1 1 11 1 2 1... nM a a a m . Để cho

gọn, ta sẽ kí hiệu ia thay cho 1ia .

Nếu 1

2n

ka am

và k là chỉ số nhỏ nhất thỏa mãn thì 2 1,i ka m i n . Điều này suy ra trực

tiếp từ tính chất không giảm của mi và chú ý rằng 1 2

2a a là một số hạng nào đó của

dãy 2 2 21 2, ,..., na a a .

Từ nhận xét trên ta thu được môt kết quả quan trọng hơn. Đặt

: | 2 | 2

: | 2 | 2k k l k k k l

k k l k k k l

S k l k m M m S k m M m

P k l k m M M P k m M M

Nếu S hoặc P có vô hạn phần tử, giả sử |S| = ∞ thì, với mỗi k ∊ S

1 1 1 1 2 2 2k k k k k

k k k km M M m dd M m M

Vì dr là dãy giảm nên nếu |S| = ∞ thì lim 0rrd

và do đó M = m.

Nếu không, thì |S|, |P| < +∞. Ta có thể giả sử |S| = |P| = 0 mà không làm ảnh hưởng đến kết

quả của bài toán. Khi đó với mọi k > 1 thì số 1

2na a không thể là số nhỏ nhất hay lớn nhất

trong dãy 1 2, ,...,k k kna a a và như vậy ta có thể xét bài toán đẹp hơn với n – 1 số, sau khi đã bỏ

đi một số 1

2na a . Bằng phương pháp quy nạp đơn giản ta thu được đpcm.

Từ bổ đề trên đây ta suy ra một kết quả trực tiếp ĐỊNH LÝ DỒN BIẾN MẠNH S.M.V Nếu : ,kf I R R I [α, β] x [α, β] x … x

[α, β],α, β ∊ R là hàm liên tục đối xứng và bị chặn dưới thỏa mãn điều kiện

1 2 1 2( , ,..., ) ( , ,..., )n nf a a a f b b b Với 1 2( , ,..., )nb b b là dãy thu được từ dãy 1 2( , ,..., )na a a theo phép biến đổi ∆, thì ta có

1 2( , ,..., ) ( , ,..., )nf a a a f a a a với 1 2 ... na a aan

.

Page 36: Chuyên đề bất đẳng thức toán học

MỘT SỐ PHƯƠNG PHÁP CHỨNG MINH BẤT ĐẲNG THỨC PHƯƠNG PHÁP DỒN BIẾN

Trang 36

Bằng định lý này, khi sử dụng dồn biến ta chỉ cần chọn ra số nhỏ nhất và số lớn nhất. Định lí về dồn biến đã được chứng minh khá chặt chẽ và có một kết quả mạnh hơn hoàn toàn bằng kiến thức sơ cấp, chúng ta hoàn toàn có thể áp dụng được.

Ngoài ra phép biến đổi ∆ có thể khác hơn, chẳng hạn thay thành 2 2

,2

a bab hoặc

bất kì một dạng trung bình nào khác. Tùy theo giả thiết của bài toàn mà ta cần chọn cách dồn biến cho phù hợp.

Ví dụ B.V.1: [IMOSL, Việt Nam đề nghị] Cho , , , 0, 1a b c d a b c d

Chứng minh rằng: 1 17627 27

abc bcd cda dab abcd (1)

Chứng minh

Đằng thức xảy ra khi 14

a b c d hoặc 1 , 03

a b c d ( và các hoán vị ).

Do đó, những đánh giá thông thường rất dễ rơi vào bế tắc.

Đặt 176, , ,27

f a b c d abc bcd cda dab abcd

Ta có:

176, , ,27

f a b c d ab c d cd cd a b

Đặt 21 176, , , , ,2 2 2 27

a b a bk f a b c d f c d a b c d cd

Nếu 176 0 0 ( , , , ) , , ,27 2 2

a b a bc d cd k f a b c d f c d

Khi đó theo định lí dồn biến mạnh ta có 1( , , , ) , , ,

4 4 4 4 27a b c d a b c d a b c d a b c df a b c d f

Đpcm.

Nếu 3176 10 , , ,27 3 27

c d a bc d cd f a b c d cd a b

Đpcm.

Trên đây là một ví dụ khá điển hình và thể hiện rõ ưu điểm của phương pháp dồn biến mạnh. Phương pháp này hữu dụng nhất khi sử dụng với bất đẳng thức 4 biến. Chúng ta sẽ xét một ví dụ sử dụng định lý S.M.V bằng cách thay số lớn nhất và 1 số bằng trung bình cộng của chúng.

Ví dụ B.V.2: Cho , , , 0a b c d và 4a b c d Chứng minh rằng:

2 2 2 2 8abc bcd cda dab abc bcd cda dab

Chứng minh

Đặt 2 2 2 2, , ,f a b c d abc bcd cda dab abc bcd cda dab

Giả sử a b c d .Xét hiệu: , , , , , ,2 2

a c a cf b d f a b c d

Page 37: Chuyên đề bất đẳng thức toán học

MỘT SỐ PHƯƠNG PHÁP CHỨNG MINH BẤT ĐẲNG THỨC PHƯƠNG PHÁP DỒN BIẾN

Trang 37

2 22 2 2 2

22 2

22 2

4 2 02

a c a cb d ac b d b d

a b b d abcd b d

(do 2 2abcd b d )

Vì vậy , , , , , , .2 2

a c a cf a b c d f b d

Theo định lí , ta chỉ cần chứng minh bất đẳng thức

với 4, 4 3 ,03

a b c x d x x

Dễ dàng chứng minh: 4 3 228 16 12 8 0x x x với 403

x (Sử dụng đạo hàm)

Bất đẳng thức được chứng minh. Đẳng thức xảy ra khi và chỉ khi 1a b c d

Ví dụ B.V.3: [Bất đẳng thức Tukervic] Cho , , , 0a b c d . Chưúng minh rằng: 4 4 4 4 2 2 2 2 2 2 2 2 2 2 2 22a b c d abcd a b b c c d d a a c b d

Chứng minh Giả sử a b c d . Xét : 4 4 4 4 2 2 2 2 2 2 2 2 2 2 2 2, , , 2f a b c d a b c d abcd a b b c c d d a a c b d

4 4 4 4 2 2 2 2 2 2 2 2, , , 2f a b c d a b c d abcd a c b d a c b d

2 2 2 2, , , , , , 0f a b c d f ac b ac d a c a c b d

Do đó theo định lí S.M.V, ta chỉ cần chứng minh bất đẳng thức khi a b c t Khi đó Bất đẳng thức đã cho 4 4 3 4 2 2 4 3 3 2 23 2 3 3 3t d t d t t d d t d t d t d Hiển nhiên đúng theo bất đẳng thức AM-GM. Đẳng thức xảy ra a b c d hoặc , 0a b c d và các hoán vị.

Ví dụ B.V.4: Cho , , , 0a b c d và 4a b c d .Chứng minh rằng:

2 2 2 21 1 1 1 1 1 1 1a b c d a b c d

Chứng minh Đặt 2 2 2 2, , , 1 1 1 1 1 1 1 1f a b c d a b c d a b c d

Và giả sử a b c d .Xét hiệu: , , , , , ,2 2

a c a cP f a b c d f b d

Ta sẽ chứng minh 0P .Thật vậy, dễ thấy 2a c nên

2 22

22 2 411 1 1 02 2 16

a c aca ca c a c

Mà theo bất đẳng thức AM-GM thì: 2

1 1 12

a ca c

.

Page 38: Chuyên đề bất đẳng thức toán học

MỘT SỐ PHƯƠNG PHÁP CHỨNG MINH BẤT ĐẲNG THỨC PHƯƠNG PHÁP DỒN BIẾN

Trang 38

Do đó: , , , , , ,2 2

a c a cf a b c d f b d

Vậy ta chỉ cần chứng minh bất đẳng thức với , 4 3a b d x c x :

Ta có 3 2 32, , ,4 3 1 1 4 3 1 5 3f x x x x x x x x

6 4 2 2 3 23 3 1 9 24 17 3 3 1 5 3x x x x x x x x x 8 7 6 5 4 3 29 24 44 72 81 68 54 36 12x x x x x x x x

26 5 4 3 29 6 23 2 18 12 12 1x x x x x x x

2 2 2 24 4 2 23 1 2 5 2 1 10 3 2 1 0x x x x x x x x

Vậy Bất đẳng thức được chứng minh. Đẳng thức xảy ra khi và chỉ khi 1a b c d

Ví dụ B.V.5: Cho , , , 0x y z t thoả mãn 4x y z t . Chứng minh rằng:

1 3 1 3 1 3 1 3 125 131x y z t xyzt

Chứng minh Đặt: , , , 1 3 1 3 1 3 1 3 131f x y z t x y z t xyzt

Không mất tính tổng quát ta giả sử x y z t . Khi đó:

Vì x y z t nên 2y t . Do đó dễ thấy: 9 1 3 1 3 131y t yt

Vậy ta có , , , , , ,2 2

x z x zf x y z t f y t

. Theo định lí S.M.V thì bất đẳng thức sẽ được

chứng minh xong nếu 1 4 3x y z a t a Thật vậy, khi đó cần chứng minh

2 231 3 1 3 4 3 125 131 4 3 1 3 4 50 28 0a a a a a a a

Dễ thấy bất đẳng thức nàu hiển nhiên đúng. Vậy BĐT đã cho được chứng minh. Dấu “=” xảy ra khi 1x y z t

hoặc 4 , 03

x y z t và các hoán vị

B.VI. Dồn biến bằng hàm lồi Hàm lồi là một công cụ cơ bản để dồn biến các bát đẳng thức “dạng cổ điển”. Để bạn đọc tiện theo dõi chúng rôi xin nhắc lại các kiến thức cơ bản sau đây: 1. Định nghĩa: Một hàm số : ,f a b được gọi là lồi nếu:

1 1 , , , , 0,1f tx ty tf x t f y x y a b t

2. Tính chất: 2.1 Giả sử f là hàm có đạo hàm đến cấp 2 trên khoảng (a,b). Khi đó f lồi trên [a,b] khi và chỉ khi ''( ) 0, ,f x x a b .

Page 39: Chuyên đề bất đẳng thức toán học

MỘT SỐ PHƯƠNG PHÁP CHỨNG MINH BẤT ĐẲNG THỨC PHƯƠNG PHÁP DỒN BIẾN

Trang 39

2.2 Nếu hàm f lồi trên [a,b] thì f liên tục trên [a,b]. Ngược lại , nếu f liên tục trên [a,b] thì f

lồi trên [a,b] , , ,

2 2

f x f yx yf x y a b

2.3 Bất đẳng thức Jensen: Giả sử f là hàm số lồi trên [a,b]. Khi đó ta có

(i) 1 21 2

1 2

......, , ,..., ,nn

n

f x f x f xx x xf x x x a b

n n

(ii) Với xi là n số thuộc đoạn [a,b] và i

là n số không âm có tổng bằng 1 ta có:

1 1 2 2 1 1 2 2... ...

n n n nf x x x f x f x f x

Các bạn có thể thấy tính chất 2.2 và mở rộng ra là bất đẳng thức Jensen có ứng dụng trực tiếp vào phương pháp dồn biến đặc biệt là các bất đẳng thức cổ điển. Một số các bài toán khó với các dạng toán này là sự kết hợp giữa phương pháp hàm lồi và các phương pháp khác trên các miền xác định khác nhau. Chúng ta sẽ cùng sử dụng hàm lồi để chứng minh một bất đẳng thức cổ điển, đó là bất đẳng thức Cauchy

Ví dụ B.VI.1: Cho n số thực dương xi. Chứng minh rằng:

1 21 2

... ...n nn

x x x x x xn

Chứng minh Lôgarit 2 vế. Bất đẳng thức cần chứng minh tương đương với

1 21 2 ln ln ... ln...ln nn x x xx x xn n

Hàm số ( ) ln( )f x x đi từ khả vi 2 lần và 2'' 0, 0f x x x . Do vậy hàm

g x f x sẽ thỏa '' 0, 0g x x . Vậy g là hàm lồi. Từ đó áp dụng BĐT Jensen ta có

ngay điều phải chứng minh. Băng cách chứng minh tương tự và áp dụng định lý 2.3 (ii) ta cũng dễ dàng chứng minh được bất đẳng thức Cauchy dạng suy rộng.

Ví dụ B.VI.2: (Vô địch Ba Lan 1992) Cho x,y,z thỏa mãn x + y + z = 1. Chứng minh rằng

2 2 2

91 1 1 10

x y zx y z

(1)

Chứng minh

Xét 2 1tf t

t

, khi đó (1) 3

3x y xf x f y f z f

Ta sẽ chứng minh f lõm hay – f lồi. Ta có

2

32

2 3''

1

t tf t

t

nên ''( ) 0, 0, 3f t t .

Vậy nếu , , 0, 3x y z thì bài toán được giải quyết.

Page 40: Chuyên đề bất đẳng thức toán học

MỘT SỐ PHƯƠNG PHÁP CHỨNG MINH BẤT ĐẲNG THỨC PHƯƠNG PHÁP DỒN BIẾN

Trang 40

Trong trường hợp còn lại thì chắc chắn ta sẽ có dấu bất đẳng thức thực sự. Do vậy cứ việc chia thành nhiều trường hợp con để xét. Để đơn giản, ta giả sử x y z .

Do 1x y z và x, y, z không đồng thời 0,1 nên z < 0, suy ra 0f z .

Nếu 12

y thì 2 2 2

1 2 901 1 1 2 5 10

x y zx y z

nên ta chỉ xét khi 12

y

Nếu 102

z thì cùng với 12

y ta có

2 22 2 2

4 4 91 1 1 5 5 101 11 1

2 2

x y z x y zx y z

, do đó, ta chỉ cần xét khi 12

z

Nếu 1 32

z thì: 2 2 2

1 1 3 7 91 1 1 2 2 10 10 10

x y zx y z

Nếu 3z thì: 2 1 4x x y z nên x ≥ 2 và do đó

2 2 2

2 1 901 1 1 5 2 10

x y zx y z

Vậy ta có điều phải chứng minh.

Tất nhiên lời giải trên chưa phải là ngắn gọn so với nhiều lời giải khác cho bài toán này mà chúng tôi được biết. Tuy nhiên tư tưởng của nó hoàn toàn trong sáng. Ở đây, nếu thay vì mong muốn dồn biến toàn cục (dồn 1 lần 3 biến) bằng việc hi vọng hợp lý hơn là dồn được 2 biến về bằng nhau thì lời giải sẽ ngắn hơn. Thật vậy, nếu có 2 trong 3 biến x, y, z thuộc đoạn 0, 3

thì dùng hàm lồi ta dồn được 2 biến này về bằng nhau, và bài toán chỉ còn 1 biến, xem như giải quyết xong. Trong phần còn lại thì việc chia trường hợp sẽ đơn giản hơn. Như vậy, chúng ta có thêm một kĩ thuật để dồn 2 biến về bằng nhau là sử dụng hàm lồi.

Mặc dù đây là một công cụ tốt, nhưng một điểm yếu rất dễ nhận ra là trong BĐT, các biến phải nằm trong các biểu thức độc lập nhau (để có thể viết thành dạng f(x1) + ...+ f(xn)). Trong khi đó, những BĐT mà ta đã gặp phần lớn không có điều đó, và ta sẽ phải làm việc với dạng tổng quát hơn là f(x1, ..., xn). Chúng ta sẽ phải thiết lập các kết quả về dồn biến cho dạng tổng quát này ở mục sau.

Kỹ thuật dồn biến bằng hàm lồi cũng có thể được áp dụng trong việc dồn biến ra biên

Định lý: Cho : ,f a b là một hàm lồi, Khi đó max , , ,f x f a f b x a b

CHỨNG MINH Vì f liên tục nên f đạt giá trị lớn nhất tại 0 ,x a b

Xét khi 0 0 1 0| | | | 2 ,x a x b x x a a b .

Theo định nghĩa hàm lồi ta có: 11 02 2

2a xf a f x f f x

0 1 0max ,f x f a f x f a f x . Với 0 0| | | |x a x b chứng minh tương tự.

Sau đây là một ví dụ áp dụng

Page 41: Chuyên đề bất đẳng thức toán học

MỘT SỐ PHƯƠNG PHÁP CHỨNG MINH BẤT ĐẲNG THỨC PHƯƠNG PHÁP DỒN BIẾN

Trang 41

Ví dụ B.VI.3: (USA TST 2004) Cho a ≥ b ≥ c ≥ 0. Chứng minh rằng

232 3a c a b c abc

Chứng minh

Đặt 233 2f b a b c abc a c

3

32 2

2.' 1 '' 03

ac abcf b f bb b

nên f là hàm lồi. Áp dụng định lý trên ta có

max ,f b f a f c . Mặt khác

23 3 3 32 2 2 22 3 2 4 0f a a c a c a c c a c a c a c ac (BĐT

Cauchy)

23 3 3 32 2 2 22 3 2 4 0f c a c ac a c a ac ac ac ac (BĐT

Cauchy) Vậy bài toán được chứng minh. Dấu bằng xảy ra khi a = b = c Tổng quát hóa bài toán trên, ta được một kết quả khá thú vị mà cách chứng minh hoàn toàn tương tự: Cho 1 2 ... 0na a a . Chứng minh rằng:

2

1 1 2 1 21 ... ...nn n nn a a a a a n a a a

Ví dụ B.VI.4: Cho , , , ,0a b c p q p q . Tìm giá trị lớn nhất của

a b cSb c a c a b

Chứng minh

2 2

1'x y z y zf x f xz y x z x y y z x z x y

3 32 2''( ) 0y zf x

x z x y

, , , ,0x y z p q p q

Vậy f là hàm lồi. Từ định lý ở trên ta dễ thấy S đạt cực trị khi bộ số (a, b, c) có k số bằng p và 3-k số bằng q. Theo BĐT Nesbit thì S ≥ 3/2. Mà với k = 0 hay 3 thì S = 3/2, vì vậy S sẽ đạt cực trị khi k = 1 hoặc k = 2. Đặt S1 = S khi k = 1 và S2 = S khi k = 2. Ta có

1 2

2 3 2 2 3 2

3

2 22 2

4 4

0

S Sq p p q

p q q p q pq p p p q p q q q p

q p

Luôn đúng do q ≥ p

Vậy giá trị lớn nhất của S là 22

p qp q p

khi a, b, c có 2 số = p và 1 số = q.

Page 42: Chuyên đề bất đẳng thức toán học

MỘT SỐ PHƯƠNG PHÁP CHỨNG MINH BẤT ĐẲNG THỨC PHƯƠNG PHÁP DỒN BIẾN

Trang 42

B.VII. Dồn biến không xác định - UMV Định lí UMV: Cho 1 2, ,..., / 0, 1, 2,.., ,n

n iD x x x x x i n D đóng và bị chặn. Gọi là tậ hợp

các phần tử trong D có t thành phần bằng 0 và n t thành phần bằng nhau 0t .

Xét 2 phép biến đổi 1 2, :T T D D như sau: Với mỗi phần tử

1 2, ,..., \na a a a D , chọn ra hai chỉ số sao cho min 0, 1, 2,..,i ta a t n và

1 2ax , ,...,j na m a a a ,sau đó thay ,i ja a bởi , ,i ja a ( ứng với 1T ) và

i ja a ( ứng với 2T ).

:f D liên tục thoả mãn: 1 2min , ,f a f T a f T a a D

Khi đó ta có min ,y

f x f y x D

Ví dụ B.VII.1: Cho , , 0a b c .Chứng minh rằng:

3 3 3 9 4 8 , ,a b c abc a b c ab bc ca (1)

Chứng minh Đặt 3 3 3, , 9 4 8 , ,f a b c a b c abc a b c ab bc ca

Đầu tiên, chúng ta thử dồn hai biến bằng nhau, xét hiệu:

2

, , , , 3 3 9 82 2 4

b cb c b cf a b c f a b c a

Rõ ràng chứng ta chưa thể có kết luận gì. Chúng ta tiếp tục thử dồn một biến về 0: , , , ,0 3 3 9 8f a b c f a b c b c a bc

Rõ ràng, biểu thức 3 3 9 8b c a hoặc âm, hoặc không âm, hay là ta có:

, , min , , ; , ,02 2

b c b cf a b c f a f a b c

Áp dụng định lí U.M.V: , , min , , ; , ,0 ; ,0,0f a b c f x x x f y y f z

với ; ;3 2

a b c a b cx y z a b c

Mặt khác: 23 3 2,0,0 4 0; , , 12 24 12 12 1 0f z z z f x x x x x x x x

23 2, ,0 2 8 8 2 2 0 , , 0.f y y y y y y y f a b c

Vậy bất đẳng thức được chứng minh, dấu “=” xảy ra , , 0,0,0 ; 1,1,1 ; 2,2,0a b c

Các bạn có thể nhận thấy rõ ràng ý nghĩa của UMV qua bài toán trên, khi mà việc hai kiểu dồn biến thôg thường là dồn hai biến bằng nhau và đẩy biến ra biên không đạt hiệu quả. Nhưng sự liên hệ giữa 2 kĩ thuật này đã giúp chúng ta sử dụng UMV một cách hết sức hiệu quả.

Page 43: Chuyên đề bất đẳng thức toán học

MỘT SỐ PHƯƠNG PHÁP CHỨNG MINH BẤT ĐẲNG THỨC PHƯƠNG PHÁP DỒN BIẾN

Trang 43

Ví dụ B.VII.2: Cho , , , 0a b c d thoả mãn 4a b c d . Chứng minh rằng:

2 4a b c d abc bcd cda dab

Chứng minh

Đặt , , , 2 4f a b c d a b c d abc bcd cda dab

Xét các hiệu:

2

, , , , , , 2 22 2 4

a ba b a bf a b c d f c d a b a b c d ab

2 2

28

4 42

a b a bc d c d X

a b a b a b

, , , ,0, ,f a b c d f a b c d

42 a b a b c d ab c d ab Y c dab a b a b

Dễ dàng chứng minh: 2

4 8

2ab a b a b a b a b a b

Hay Y X nên , , min , , , ; ,0, ,2 2

c d Y a b a bf a b c f c d f a b c dc d X

Áp dụng định lí UMV ta có:

4 4 4, , , min 1,1,1,1 ; , , ,0 ; 2, 2,0,0 ; 4,0,0,0 03 3 3

f a b c d f f f f

Đẳng thức xảy ra , , , 1,1,1,1 ; 4,0,0,0a b c d .

Ví dụ B.VII.3: Cho 1 2, ,..., 0na a a thoả mãn 1 2 ... .na a a n Tìm

2 2 21 2 1 2

1 2

1 1 1min ... ... ...n nn

S a a a a a aa a a

Chứng minh

Đặt 2 2 21 2 1 2 1 2

1 2

1 1 1, ,..., ... ... ...n n nn

f a a a a a a a a aa a a

Xét các hiệu:

1 2 1 2 3 1 2 3 43 4

1 1 1, ,..., 0, , ,..., 2 ... ...n n nn

f a a a f a a a a a a a a aa a a

(*)

1 2 1 21 2 3, ,..., , , ,...,

2 2n na a a af a a a f a a

Page 44: Chuyên đề bất đẳng thức toán học

MỘT SỐ PHƯƠNG PHÁP CHỨNG MINH BẤT ĐẲNG THỨC PHƯƠNG PHÁP DỒN BIẾN

Trang 44

21 2

3 43 4

1 1 12 ... ...4 n

n

a aa a a

a a a

(**)

1 2 1 21 2 1 2 3 3, ,..., min 0, , ,..., , , , ,...,

2 2n n na a a af a a a f a a a a f a a

Do vậy, áp dụng định lí UMV ta có: 1 2 1,, ,..., min , ,..., ,0,0,...,0n k n

n n nf a a a fk k k

Hay giá trị nhỏ nhất của S là: 12 2

min 2 , ,1 1 2

nn n nnn n n

B.VIII. Dồn biến toàn miền (EMV) Ở tất cả các phép dồn biến trước, chúng ta thấy rằng các biến đều có một đại lượng

không đổi, thường là tổng, tổng bình phương, tổng hoán vị hay tích… Phương pháp dồn biến toàn miền cũng có một đại lượng không đổi, nhưng đại lượng đó lại hoàn toàn mới, đó là a – b , b – c , c – a

Tư tưởng chủ đạo của phương pháp dồn biến toàn miền là chứng minh khi tăng hoặc giảm các biến đi cùng một đại lượng, bất đẳng thức yếu đi hoặc không đổi. Từ đó chỉ cần chứng minh bất đẳng thức với 1 biến ở biên. Điều này đặc biệt hữu dụng với các bất đẳng thức hoán vị.

Khi dồn biến toàn miền trong bất đẳng thức n biến, với 1 biến ở biên, ta còn phải chứng minh bất đẳng thức với n – 1 biến khá khó khăn. Vì vậy ứng dụng chủ yếu của dồn biến toàn miền là trong bất đẳng thức 3 biến.

B.VIII.1 . EMV với biên tại 0:

Ví dụ B.VIII.1: Chứng minh với mọi số thực không âm a, b, c ta luôn có 3 3 3 3 4a b c abc a b b c c a

Chứng minh Bất đẳng thức tương đương với

2 2 2 8a b c a b b c c a a b b c c a (1)

Không mất tổng quát giả sử c = min {a, b, c}. Thay a = a – c, b = b – c, c = 0 thì a – b, b – c, c – a không đổi còn a + b + c giảm đi. Vậy vế trái của (1) giảm đi còn vế phải không đổi. Do đó ta chỉ cần chứng minh bài toán trong trường hợp a, b ≥ c = 0. Khi đó bất đẳng thức tương đương với

3 3 4a b ab b a

Điều này hiển nhiên đúng vì 2 2 2 3 3 34 4 4 4ab a b a b a b ab b a b b a

Đẳng thức xảy ra khi a = b = c. Bài toán trên đơn giản nhưng thể hiện rất rõ tư tưởng của phép dồn biến toàn miền. Một cách hoàn toàn tương tự ta cũng chứng minh được bài toán với 4 biến Chứng minh với mọi a, b, c, d không âm

Page 45: Chuyên đề bất đẳng thức toán học

MỘT SỐ PHƯƠNG PHÁP CHỨNG MINH BẤT ĐẲNG THỨC PHƯƠNG PHÁP DỒN BIẾN

Trang 45

4 4 4 4 4 2a b c d abcd a b b c c d d a a

Ví dụ B.VIII.2: Cho a, b, c ≥ 0 thỏa mãn a + b + c = 3. Chứng minh: 2 2 2 4a b b c c a abc

Chứng minh Trước tiên, muốn sử dụng kỹ thuật dồn biến toàn miền, ta phải loại bỏ điều kiện bằng cách thuần nhất hóa bất đẳng thức hay đưa bài toán trở thành:

Cho a, b, c ≥ 0. Chứng minh rằng 32 2 2 427

a b b c c a abc a b c

Điều này

3

2

8 6 3 27 0

( , , ) 4 4 27 0

a abc b b c bc c b

f a b c a b c b c a b b c c a

Rõ ràng ta có: ( , , ) ( , , ), 0, min , ,f a b c f a x b x c s x a b c

Vì vậy ta chỉ càn chứng minh bất đẳng thức khi c = 0: 32 427

a b a b

Áp dụng bđt Cauchy ta có:

3

32 42 24 . . 42 2 3 27

a a ba aa B b a b

Bất đẳng thức được chứng minh Dấu = xảy ra khi a = b = c = 1 hoặc a = 2, b = 1, c = 0 và các hoán vị.

Ví dụ B.VIII.3: Cho a, b, c ≥ 0 đôi một phân biệt. Chứng minh rằng 2 2 2a b b c c a a b b c c a

b c c a a b b c c a a b

(1)

Chứng minh Giả sử c = min{a, b, c} và đặt

2 2 2

, , a b b c c a a b b c c af a b cb c c a a b b c c a a b

Ta có:

2c a c b a ba b b c c ab c c a a b b c a b c a c b

Vậy khi giảm mỗi biến đi 0,x c do a – b, b – c, c – a không đổi nên , ,f a b c giảm.

Vậy ta chỉ còn chứng minh bất đẳng thức với c = 0, tức là 2 2 2a b b a a b b a

b a a b b a a b

Chuẩn hóa b = 1, ta cần chứng minh: 2

22

1 11 11 1

a aa aa a a a

Khảo sát hàm số trên ta sẽ có điều phải chứng minh B.VIII.2 . EMV với biến trong tam giác

Page 46: Chuyên đề bất đẳng thức toán học

MỘT SỐ PHƯƠNG PHÁP CHỨNG MINH BẤT ĐẲNG THỨC PHƯƠNG PHÁP DỒN BIẾN

Trang 46

Ví dụ B.VIII.4: Cho a, b, c là độ dài cạnh của một tam giác (có thể suy biến). Tìm giá

trị lớn nhất của biểu thức: , , a b cf a b cb c c a a b

Chứng minh Giả sử a = max{a,b,c}. Ta có biến đổi đơn giản sau:

23, ,

2 2b c

f a b ca b a c

Dễ thấy , , , , , 0,f a b c f a x b x c x x k . Trong đó k là đại lượng thích hợp mà

chúng ta sẽ tìm. Từ đó chỉ cần chứng minh bất đẳng thức trong trường hợp một biến ở biên. Thử cho x = k = min {a,b,c} tức là cho 1 biến = 0, chẳng hạn c = 0.

Khi đó , ,0 a bf a bb a

. Không thể kết luận gì về max trong trường hợp này. Đó là do khi

cho c = 0, ta cũng cho luôn a = b (do a, b, c là 3 cạnh tam giác). Vì vậy , ,f a b c yếu đi rất

nhiều. Chú ý rằng, do a = max {a, b, c} nên điều kiện cần và đủ để a, b, c là 3 cạnh tam giác (có thể suy biến) là a ≤ b+c. do vậy ta nhận thấy ngay b + c chính là biên của biến a. từ đó ta sẽ đẩy a về b + c. Cho x = b + c – a ≥ 0. Khi đó

3, , 1 2 2

2 2 2 2b c bcf b c b c

c b b c b c c b

Đẳng thức xảy ra khi a = b, c = 0 và các hoán vị. Vậy max f(a, b, c) = 2.

Ví dụ B.VIII.5: Cho a, b, c là độ dài 3 cạnh tam giác. Chứng minh ràng

4 3 3a b c a c bb c a c b a

(1)

Chứng minh

2 2 2

(1) 7 6 0

7 0

a b c a c b a c b a b cb c a c b a c b a b c a

a b b c c a a b c b c a c a b

Nếu cùng giảm a, b, c đi x ≥ 0 thì vế trái giảm.Vì vậy như bài trên, ta chỉ cần chứng minh bất đẳng thức với c = a + b, tức là

2 3 3 3 3 2 2

22 3

7 0 2 2 7 7 0

7 72 2 0.4 8

ab b a a b a b a b a b ab a b

b b a a b a

Bất đẳng thức được chứng minh. Đẳng thức xảy ra khi chỉ khi a = b = c

Qua các ví dụ trên, có thể thấy rằng cái khó của EMV chính là biến đổi bất đẳng thức về dạng chứa các biến a – b, b – c, c – a phù hợp. Việc này đòi hỏi một quá trình rèn luyện lâu dài. Tuy nhiên một khi đã biến đổi xong. Việc áp dụng EMV trở nên khá dễ dàng.

Page 47: Chuyên đề bất đẳng thức toán học

MỘT SỐ PHƯƠNG PHÁP CHỨNG MINH BẤT ĐẲNG THỨC PHƯƠNG PHÁP DỒN BIẾN

Trang 47

B.IX. Định lí dồn biến tổng quát {GMV – GENERAL MIXING VARIABLES}

Trong mục này, chúng tôi sẽ giới thiệu định lí GMV dùng để dồn biến cho n số. Đây là một định lí rất tổng quát, bao gồm gần như toàn bộ các khả năng của dồn biến.

1. Chúng ta bắt đầu bằng một số định nghĩa trong không gian n . Định nghĩa 1:

Không gian n là tập hợp các bộ thứ tự 1 2, ,..., nx x x x với , .nix i

Một dãy 1, 2, ,, ,...,m m m n mx x x x trong n gọi là hội tụ về 1 2, ,..., nnz z z z nếu

từng dãy ,i mx hội tụ về iz khi , 1, 2,...,m i n .

Cho .nD Một hàm số :f D gọi là liên tục trên D nếu : Với mọi dã

mx D và với mọi z D sao cho mx hội tụ về z thì ta đều có: mf x hội tụ về

f z

Định nghĩa 2: Cho nD . Ta nói: D đóng nếu với mọi dãy và với mọi nz sao cho mx hội tụ về z thì ta đều có

z D . D bị chặn nếu tồn tại số thực M sao cho: 1 2, ,..., nx x x x D

thì , 1, 2,...,ix M i n .

Định lý Welerstrass: Cho D đóng và bị chặn trong n và : nf D liên tục. Thì f đạt

giá trị nhỏ nhất trên D , nghĩa là tồn tại 0x D sao cho: 0 , .f x f x x Chúng ta

cũng có những kết quả tương tự với giá trị lớn nhất.

Bình luận: Định lý này là một mở rọng của một kết quả khá quen thuộc: “Cho ,a b là 1 khoảng đóng trong và : ,f a b liên tục thì f có giá trị nhỏ nhất trên ,a b ”. Do đó, về mặt trực giác thì định lí 1 khá rõ ràng.

2. Định lý GMV: Ta sẽ luôn giả thiết rằng: D là một tập con trong n , và là một tập con đóng của D . :f D là một hàm số liên tục sao cho f có giá trị nhỏ nhất trên .

1 2, ,..., :kT T T D D là các phép biến đổi sao cho 1 ... ,kT x T x x x

Ta sẽ đặt ra các tiêu chuẩn để giá trọ nhỏ nhất của f trên cũng chính là giá trị nhỏ nhất của f trên D . Ta có: Định lý GMV1: Nếu

1,

min , \jj kf x f T x x D

1, ,j k x D ta có lim mjm

T x

, trong đó 0 1, m mj j j jT x x T x T T x

Thì min ,y

f x f y x D

.

Định lý GMV2: Nếu

Page 48: Chuyên đề bất đẳng thức toán học

MỘT SỐ PHƯƠNG PHÁP CHỨNG MINH BẤT ĐẲNG THỨC PHƯƠNG PHÁP DỒN BIẾN

Trang 48

D đóng và bị chặn trong n .

1,min , \jj k

f x f T x x D

Thì min ,y

f x f y x D

, hơn nữa đăng thức không xảy ra trên \D .

Định lý GMV3: Nếu D đóng và bị chặn trong n .

1,min , \jj k

f x f T x x D

Tồn tại các hàm số jh liên tục thoả mãn: , \j j jh x h T x x D .

Thì min , .y

f x f y x D

Chứng minh GMV1: Lấy tuỳ ý. Ta có: 1,

min jj kf x f T x

và bằng quy nạp suy ra

1,

min ,mjj k

f x f T x x D

. Do f liên tục và lim mmT x

nên:

1,

1,

lim min min lim minm mj jm m yj k

j k

f x f T x f T x f y

Hơn nữa, nếu 1,

min jj kf x f T x

thì do minj y

f T x f y

nên miny

f x f y

Chứng minh GMV2: Do định lý Weierstrass, tòn tại 0x D sao cho 0 ,f x f x x D .

Nếu 0x thì 0 01,min jj k

f x f T x

, mâu thuẫn. Vậy 0x và ta có Đpcm.

Chứng minh GMV3: Lấy 0y sao cho 0 miny

f y f y

. Giả sử phản chứng rằng tồn

tại z D sao cho 0 .f z f y Do jh x bị chặn dưới trên D nên bằng cách cộng thêm

một hằng số khi cần, ta có thể giả sử 0, , 1, 2,...,jh x x D j k .

Chọn 0 đủ nhỏ ta có: 01

k

jj

f z h z f y

Đặt 1,

min ,jj kg x f x h x x D

thì :g D liên tục và hơn nữa

1,

min , \jj kg x g T x x D

và 0 min

yg z f y g y

.

Điều này mâu thuẫn với định lí 2, hay giả thiết phản chứng là sai, suy ra đpcm.

Bình luận: Tuy hình thức páht biểu ngắn gọn nhưng GMV có tầm ứng dụng cực kì rộng rãi. Cứ mỗi một ( hay một vài ) phép biến đổi T thích hợp là ta lại có một tiêu chuẩn dồn biến. Chẳng hạn, ta có ngay hai hệ quả sau đây.

Hệ quả 1:[Phạm Kim Hùng, SMV – Strongly Mixing Variables] Cho: nD R là một tập đóng, bị chặn và 0 , ,...,s s s s D .

Phép biến đổi T như sau: Với mỗi bộ n số 1 2, ,..., na a a , ta chọn ra số lớn nhất và số

nhỏ nhất rồi thay thế bằng trung bình cộng của chúng. Giả sử rằng :T D D . :f D là một hàm số liên tục, đối xứng thoả mãn: ,f a f T a a D .

Page 49: Chuyên đề bất đẳng thức toán học

MỘT SỐ PHƯƠNG PHÁP CHỨNG MINH BẤT ĐẲNG THỨC PHƯƠNG PHÁP DỒN BIẾN

Trang 49

Khi đó: 0 ,f a f s a D

Chứng minh: Chọn 21 2

1, ,...,

n

n ii

h a a a a

thì dễ thấy :h D liên tục và

0, \h a h T a a D s . Áp dụng định lí GMV3 ta có đpcm.

Ghi chú: Ta cũng có thể áp dụng GMV1, khi dó chỉ cần kiểm tra với mọi a D thì 0lim m

mT a s

.

Sự kiện này khá rõ ràng về mặt trực giác, và đề nghị bạn đọc tự chứng minh. Với cách làm na, chúng ta có thể bỏ qua giả thiết T đóng và bị chặn (nghĩa là Hệ quả 1 ở trên đúng với mọi tập con D của n sao cho T D D )

Hệ quả 2: [Đinh Ngọc An, UMV – Unđefine Mixing Variables] Cho:

1 21

, ,..., 0, onstn

nn i i

iD x x x x x x c

. Gọi là tậ hợp có t thành phần

bằng 0 và n t thành phần bằng nhau 0t .

:f D liên tục, đối xứng thoả mãn:

1 2 1 21 2 3 1 2 3, ,..., min , , ,..., , 0, , ,...,

2 2n n na a a af a a a f a a f a a a a

Thì min ,y

f x f y x D

Chứng minh: Chọn 2 phép biến đổi 1 2, :T T D D như sau: 2 2 ,T a T a a a và

với mỗi phần tử 1 2, ,..., \na a a a D , chọn ra 2 chỉ số Þi sao cho

min 0 1, 2,...,i ta a t n avf 1 2ax , ,...,j na m a a a , ứng với 1T ta thay ,i ja a bởi trung

bình cộng của chúng, ứng với 2T ta thay ,i ja a bởi 0, i ja a . Thì ta có

1 2min , ,f a f T a f T a a D

Chọn 21 1 2

1, ,...,

n

n ii

h a a a a

và thì 1 2, :h h D liên tục và

0, \ , 1, 2,...j j jh a h T a a D s j n . Áp dụng GMV3 ta có đpcm.

Ghi chú: Ta cũng có thể kiểm tra với mọi a D thì 20lim 0,...,0, ,...,m

mT a r r

rồi áp dụng GMV1. Do đây là một phần khá khó nên ta sẽ chỉ xét một ví dụ sau:

Ví dụ B.IX.1: (Bất đẳng thức Cauchy) Cho n số thực không âm x1,x2,…,xn. Chứng minh rằng

1 2 1 2... ...nn nx x x n x x x

Chứng minh Chuẩn hóa 1 2... 1nx x x và chứng minh 1 2 ... nx x x n .

Tất nhiên ta chỉ xét khi ,ix n i

Page 50: Chuyên đề bất đẳng thức toán học

MỘT SỐ PHƯƠNG PHÁP CHỨNG MINH BẤT ĐẲNG THỨC PHƯƠNG PHÁP DỒN BIẾN

Trang 50

Xét 1 2 1 2, ,..., | 0, , ... 1n i nD x x x x x n x x x thì dễ thấy D đóng và bị chặn.

Xét 0 1,1,1...,1x . Xét :f D liên tục như sau:

Với mỗi 1 2, ,..., nx x x x D thì 1 2 ... nf x x x x

Xét : \T D D như sau: Với mỗi 1 2, ,..., \nx x x x D thì tồn tại i jx x và ta đặt

T(x) là bộ thu được từ x sau khi thay xi và xj bởi trung bình nhân của chúng. Khi đó dễ thấy

20i jf x f T x x x . Vậy ta có thể áp dụng GMV2 để suy ra

0 ,f x f x x D , hơn nữa dấu = chỉ xảy ra khi x = x0.

B.X. Bài tập Bài B.X.1: Cho , , 0a b c thoả mãn 1abc . Chứng minh rằng:

2 2 2 3 1 1 162

a b c a b ca b c

Chứng minh

Đặt 2 2 2 3 1 1 1, , 62

f a b c a b c a b ca b c

Không mất tính tổng quát ta giả sử min , ,a a b c

Xét hiệu , , , ,d f a b c f a bc bc

2

2 2 22 3 3 3 32 2 2 2

b cb c b c b c b c

bc bc

(1)

Do min , ,a a b c và 1abc nên

1bc 2 23 3 3 3 3 34, 4 1 02 2 2 2 2 2

b c b cbc bc

Nên từ (1) suy ra 0 , , , ,d f a b c f a t t (với 2, 1a t at )

Xét 2 2 3 3 1 2, , 2 6 22 2

f a t t a t a ta t

2

6 5 4 3 22 4 4

3 3 1 13 6 6 12 6 3 22 2 2t t t t t t t

t t t t

2 4 3 21 4 3 4 2t t t t t (2)

Đặt 4 3 24 3 4 2, 1,g t t t t t t

Có 3 2 34 3 1 4 22

g t t t t t x t y t

(Với 0,1 2x y )

02

t yg t

t

Xét bảng biến thiên

Page 51: Chuyên đề bất đẳng thức toán học

MỘT SỐ PHƯƠNG PHÁP CHỨNG MINH BẤT ĐẲNG THỨC PHƯƠNG PHÁP DỒN BIẾN

Trang 51

t 1 y 2 g t 2 + 0 - 0 +

g t 6

g y

5

Vậy từ Bảng biến thiên ta có 5 0 / 1,g t nên từ (2) , , 0f a t t hay

, , 0f a b c (Đpcm)

Dấu “=” xảy ra khi 1a b c Bài B.X.2: Cho , , 0, 3a b c a b c Chứng minh rằng:

3 3 3 3 3 3 3 3 3 36a b c a b b c c a ab bc ca

Chứng minh Không mất tổng quát giả sử a b c . Đặt

3 3 3 3 3 3 3 3 3, , 36f a b c ab bc ca a b c a b b c c a

Khi đó 3 33 3, ,0 36f a b c a b c a b c a b c

Mà 36 36a b c ab bc ac

33 3 3 3

33 3 3 3 3 3 3

a b c a b c

a b b c c a a b c

, , , ,0f a b c f a b c

Do đó ta chỉ cần chứng minh bất đẳng thức trong trường hợp c = 0 hay

3 3 3 3 3 3 2 236 36ab a b a b a b a b

Đặt t = ab, a + b = 3 nên bất đẳng thức cần chứng minh tương đương với

2 2 3 236 27 9 6 3 4 3t t t t t t với 90,4

t Điều này hiển nhiên đúng theo

bđt Cauchy cho 3 số không âm 3 3

, , 42 2t t

Vậy bài toán đã được chứng minh, dấu bằng xảy ra khi c = 0, ab = 2 , a + b = 3 và các hoán vị 1, 2, 0a b c và các hoán vị

Page 52: Chuyên đề bất đẳng thức toán học

MỘT SỐ PHƯƠNG PHÁP CHỨNG MINH BẤT ĐẲNG THỨC BẤT ĐẲNG THỨC SCHUR VÀ PHƯƠNG PHÁP BIẾN ĐỔI PQR

Trang 52

C BẤT ĐẲNG THỨC SCHUR VÀ PHƯƠNG PHÁP BIẾN ĐỔI

PQR C.I. Bất đẳng thức Schur

C.I.1 . Lý thuyết

a) Bất đẳng thức Schur dạng chính tắc Cho a,b,c ≥ 0, k là số thực bất kì thì ta luôn có

( )( ) ( )( ) ( )( ) 0k k ka a b a c b b c b a c c a c b CHỨNG MINH Không mất tính tổng quát giả sử 0a b c khi đó ta có VT ( )( ) ( )[ ( ) ( )] 0k k kc c a c b a b a a c b b c Điều này hiển nhiên đúng. Bất đẳng thức Schur được chứng minh Dấu “=” xảy ra a=b=c hoặc a=b,c=0 cùng các hoán vị. Các trường hợp thường gặp của Schur Với k=0

2 2 2

( )( ) ( )( ) ( )( ) 0( )( ) ( )( ) ( )( ) 0

k k ka a b a c b b c b a c c a c ba b a c b c b a c a c b

a b c ab bc ca

Với k=1 2

3 3 3

( )( ) ( )( ) ( )( ) 0( )( ) ( )( ) ( )( ) 0

3 ( ) ( ) ( )

k ka a b a c b b c b a c c a c ba a b a c b b c b a c c a c ba b c abc ab a b bc b c ca c a

Với k=2

2 2 2

4 4 4 2 2 2 2 2 2

( )( ) ( )( ) ( )( ) 0( )( ) ( )( ) ( )( ) 0

( ) ( ) ( ) ( )

k k ka a b a c b b c b a c c a c ba a b a c b b c b a c c a c ba b c abc a b c ab a b bc b c ca c a

b) Các dạng suy rộng của bất đẳng thức Schur Định lí 1: Bất đẳng thức Schur suy rộng 1(Vornicu Schur)

Phần

C

Page 53: Chuyên đề bất đẳng thức toán học

MỘT SỐ PHƯƠNG PHÁP CHỨNG MINH BẤT ĐẲNG THỨC BẤT ĐẲNG THỨC SCHUR VÀ PHƯƠNG PHÁP BIẾN ĐỔI PQR

Trang 53

Xét bất đẳng thức ( )( ) ( )( ) ( )( ) 0x a b a c y b c b a z c a c b (1) Khi đó (1) đúng với mọi 0a b c và , , 0x y z nếu thoả mãn 1 trong các khả năng sau: 1. x y hoặc y z 2. ax by 3.bz cy (nếu a,b,c là 3 cạnh của 1 tam giác)

4. x y z

5. 2 2 2 2( )x y z xy yz xz CHỨNG MINH Nếu x y thì ta có ( )( ) ( )[ ( ) ( )] 0VT z a b b c a b x a c y b c Nếu z y thì ta có ( )( ) ( )[ ( ) ( )] 0VT x a b a c b c z a c y a b

Với 0a b c thì ( )aa c b cb

Vì vậy ta có

( )( )( )( )( ) ( )[ ( ) ( )] ( )( ) 0a b b c ax byVT z a b b c a b x a c y b c z a b b cb

Do 0a b c và a,b,c là độ dại 3 cạnh của 1 tam giác nên ( )ba c a bc

( )( ) ( )[ ( ) ( )]( )( )( )( )( ) 0

VT x a b a c b c z a c y a bb c a b bz cyx a b a c

c

4. Ta có

2

( )( ) ( )( ) ( )( ) 0( )( ) ( )( ) ( )( )

. .

( . . )

2

( )

x a b a c y b c b a z c a c bx a b a c z c a c b y b c a b

a c a cx z yb c a b

a b b cx z x z yb c a b

x z xz y

x z y

x z y

Ta có

2 2 2

2 2 2

2 2

( )( ) ( )( ) ( )( ) 0( )( ) ( )( ) ( )( ) 0( )( ) ( )[( ) ( )] ( )( ) 0

( ) ( )( )( ) ( ) 0(*)

x a b a c y b c b a z c a c bx y z a b y z x b c z x y c ax y z a b y z x a b c a z x y c a

y a b y z x c a a b z c a

Nếu y=0 thì

2 2 2 2 2

2

2( ) 2( ) 0

x y z xy yz xz x z zxz x z x

Khi đó 2(*) ( ) 0z c a (luôn đúng)

Page 54: Chuyên đề bất đẳng thức toán học

MỘT SỐ PHƯƠNG PHÁP CHỨNG MINH BẤT ĐẲNG THỨC BẤT ĐẲNG THỨC SCHUR VÀ PHƯƠNG PHÁP BIẾN ĐỔI PQR

Trang 54

Nếu 0y ta coi VT là tam thức bậc hai với biến (a - b). Để chứng minh (*)đúng ta chứng minh 0 . Thật vậy

2 2 2

2 2 2 2

0 ( ) ( ) 4 ( ) 0( ) 4 0 2( )

y z x c a yz c ay z x yz x y z xy yz xz

(Do các định lí từ định lí 2 trở đi không được dùng nhiều trong chuyên đề nên chỉ được viết mang tính giới thiệu mà không được chứng minh cụ thể.)

Định lí 2: Cho p,q,x,y,z,a,b,c là các số dương thoả mãn (a,b,c) và (x,y,z) là các bộ đơn điệu cùng chiều.Khi đó ta luôn có

[( )( ) ( )( )] [( )( ) ( )( )][( )( ) ( )( )] 0

p p q q q q p p p p q q q q p p

p p q q q q p p

x a b a c a b a c y b c b a b c b az c a c b c a c b

Các bất đẳng thức hệ quả: Cho , , , , , , 0a b c m n p q Ta có

[ ( )( ) ( )( )] 0m p p q q m q q p p

cyca a b a c a a b a c

( )( ) 0m n n m p p p p

cyca b a b c a c a

0m n n m p p p p q q q q q q q q p p q q

cyca b a b a b a c a b a c a b a c a b a b

Định lí 3: Xét bất đẳng thức sau, trong mỗi hàm số f : ta có ( ) ( ) ( ) ( ) 0f a b f a c f a d f a e

Bất đẳng thức đúng thì hàm số f phải thoả mãn 2 điều kiện sau : f là hàm đơn điệu tăng

. ( ) 0x f x x Định lí 4: Bất đẳng thức Schur mở rộng cho tam giác

Cho tam giác ABC và A1B1C1 có các cạnh lần lượt là a,b,c và a1,b1,c1.Khi đó ta có: 1 1 1( )( ) ( )( ) ( )( ) 0 1a bc a b a c b ac b c b a c ab c a c b Mệnh đề : Cho M là một điểm nằm trong tam giác ABC. Các đưòng thẳng AM, BM, CM cắt các cạnh BC, CA, AB lần lượt tại A1, B1, C1.Khi đó (a.MA.MA1, b.MB.MB1, c.MC.MC1) là độ dài 3 cạnh của 1 tam giác. Sử dụng mệnh đề trên và định lí 4 ta có các hệ quả sau

2

( )( ) 0

( )

a

cyc

a b c acyc cyc

m a b a ca

a m bc m m m

0cyc

x x y x zy z

Page 55: Chuyên đề bất đẳng thức toán học

MỘT SỐ PHƯƠNG PHÁP CHỨNG MINH BẤT ĐẲNG THỨC BẤT ĐẲNG THỨC SCHUR VÀ PHƯƠNG PHÁP BIẾN ĐỔI PQR

Trang 55

1. 0

0

0

cos 0

cyc

a acyc

cyc

cyc

MA MA a b a c

m h a b a c

IA a b a c

A a b a c

C.I.2 . Ví dụ

Ví dụ C.I.1: Bất đẳng thức Nebit 32

a b cb c a c a b

(*)

Chứng minh

3 3 3

(*) 2[ ] 3( )

3 (1)

a a b a c b b c b a c c a c b a b b c c a

a b c abc ab a b bc b c ca c a

Vì (1) luôn đúng theo bất đẳng thức Schur bậc 1 nên bất đẳng thức Nebit được chứng minh.

Nhận xét: Với bất đẳng thức Nesbit ta có rất nhiều cách chứng minh khác nhau nhưng việc quy đồng mẫu số với các bất đẳng thức 3 biến dạng đơn giản là suy nghĩ rất tự nhiên và cách chứng minh sau khi quy đồng là vô cùng đơn giản.Tuy nhiên không phải bất đẳng thức nào cũng có áp dụng bđt Schur trực tiếp như vậy.Sau đây sẽ tìm hiểu một bất đẳng thức như vậy

Ví dụ C.I.2: Cho , , 0a b c CMR 3 3 3

2 2 2 2 2 2

3( )a b c ab bc caAb bc c c ca a a ab b a b c

Chứng minh Áp dụng bất đẳng thức Bunhiacopxki ta có

22 2 2 2 2 2 2 2 2

2 2 2 2

2 2 2 2 2 2

2 2 2 2

.

( )

( )( ) ( ) 3

A a b bc c b c ca a c a ab b a b c

a b cAa b bc c b c ca a c a ab b

a b cAab a b bc b c ca c a abc

Mặt khác ta dễ dàng nhận thấy 2( ) 3( )a b c ab bc ca Nên ta cần chứng minh

2 2 2 2

22 2 2

24 4 4 2 2 2 2 2 2 2 2

4 4 4 2 2 2 2 2 2

( )( )

3

2( ) ( ) ( ) ( )

( ) ( ) (*)

a b c a b cab a b bc b c ca c a

a b c a b c ab a b bc b c ca c a abc

a b c a b b c c a abc a b c ab a b bc b c ca c a

a b c abc a b c ab a b bc b c ca c a

Page 56: Chuyên đề bất đẳng thức toán học

MỘT SỐ PHƯƠNG PHÁP CHỨNG MINH BẤT ĐẲNG THỨC BẤT ĐẲNG THỨC SCHUR VÀ PHƯƠNG PHÁP BIẾN ĐỔI PQR

Trang 56

Bất đẳng thức (*) luôn đúng theo bất đẳng thức Schur bậc 2 đpcm Dấu “=” xảy ra a=b=c

Ví dụ C.I.3: Cho , , 0a b c và 3a b c CMR 3 (*)2

a b ca bc b ac c ab

Chứng minh Ta có

3 3( ) 2 ( )2 2 2

2 ( )(*)2

2 ( ) 02

2 20

3 2 20

(

cyc

cyc cyc

cyc

cyc

cyc

ab bc ca a b c bcab bc ca ab bc ca

a a b c bca bc ab bc ca

a a b c bca bc ab bc ca

a ab bc ca a bc a b c bca bc a b c

bc a bc ab aca bc

bc a

) 2 2

0

( )( ) 0

( )( ) 0(1)( )

cyc

cyc

cyc

b c a bc ac aba bc

bc a c a ba bc

a b a ca a bc

Đặt

1 1 1, ,( ) ( )

x y za a bc b b ca c c ab

vì , , 0a b c nên , , 0x y z và ta có

(1) 0(2)x a b a c y b c b a z c a c b

Mặt khác không mất tính tổng quát ta giả sử a b c khi đó ta có

1 1c a bc b b ac

Nên (2) đúng theo định lí Schur suy rộng đpcm Dẫu “=” xảy ra a b c

Ví dụ C.I.4: Cho , , 0a b c .Chứng minh rằng 2 2 2

2 2 2 2 2 2 0a bc b ac c abb bc c c ac a a ab b

Chứng minh Ta có

Page 57: Chuyên đề bất đẳng thức toán học

MỘT SỐ PHƯƠNG PHÁP CHỨNG MINH BẤT ĐẲNG THỨC BẤT ĐẲNG THỨC SCHUR VÀ PHƯƠNG PHÁP BIẾN ĐỔI PQR

Trang 57

2 2

2 2 2 2 2 2

2

2 2 2 2 2 2

2cyc cyc cyc

cyc cyc

a bc a ac ab bc ac ab bcb bc c b bc c b bc c

a b a c ab a bb bc c a ac c b bc c

2

2 2 2 20 , , 0

cyc

ab a ba b c

a ac c b bc c

nên ta cần chứng minh

2 2 0

cyc

a b a cb bc c

(*)

Mặt khác không mất tính tổng quát ta giả sử a b c khi đó ta có

2 2 2 2 2 2 2 2

1 1 0a b a b c

b bc c a ac c a ac c b bc c

( a b c )

Nên theo định lí Schur suy rộng ta có(*) luôn đúng đpcm Dấu “=” xảy ra a b c

Ví dụ C.I.5: Cho , , 0a b c .Chứng minh rằng 3 3 3a abc b abc c abc a b cb c a c a b

Chứng minh Ta có:

3 3 3

3

2

cyc

a abc b abc c abcA a b cb c a c a b

a abc ab c

a a b a cb c a bc b c a

Đặt

2

2

2

,

,

axb c a bc b c a

bya c b ac a c b

cza b c ab a b c

Ta có A x a b a c y b c b a z c a c b

Mặt khác không mất tính tổng quát ta giả sử 0a b c thì ta có

Page 58: Chuyên đề bất đẳng thức toán học

MỘT SỐ PHƯƠNG PHÁP CHỨNG MINH BẤT ĐẲNG THỨC BẤT ĐẲNG THỨC SCHUR VÀ PHƯƠNG PHÁP BIẾN ĐỔI PQR

Trang 58

2 2

2 2

2 2

2 2 2

2 2

0

0

0(*)

x y

a b ac c a c a ab b a bc b c b c bc

a b ac c a b a bc b ca b ab

a b ac c a b a bc b c

a b c a ab b c a b abca b ab

a b ac c a b a bc b c

Vì 0a b c nên 2 2 2 2 0c a ab b abc c ab ab abc abc

* luôn đúng

Nên theo định lí Schur suy rộng ta có đpcm Dấu “=” xảy ra a b c hoặc 0a b , 0c cùng các hoán vị

Ví dụ C.I.6: Cho , , 0a b c CMR

3 3 3

3 3 3

32

a abc b abc c abcb c a c a b

Chứng minh

Vì 32

a b cb c a c a b

nên ta cần chứng minh

3 3 3

3 3 3

20

cyc

a abc b abc c abc a b cb c a c a bb c a c a b

a a b a cb c b c a bc a b c

Không mất tính tổng quát giả sử a b c khi đó ta có

2 2 2 0

. . (1)

b c a a b c a b ab c

c a b b c a

c a c a b c a b c b c a b c

Và ta có

2 22 2 2 2 2

2 2

2 2

0

. . (2)

c a b ac b c a bc c a b a b c ac bc ab

c a b ac b c a bc

c a c a b ac b c b c a bc

Từ (1) và (2)

2 2

2 2

b c b c a bc a b c c a c a b ac b a c

a b

b c b c a bc a b c c a c a b ac b a c

theo định lí Schur suy rộng bất đẳng thức (*)được chứng minh

Page 59: Chuyên đề bất đẳng thức toán học

MỘT SỐ PHƯƠNG PHÁP CHỨNG MINH BẤT ĐẲNG THỨC BẤT ĐẲNG THỨC SCHUR VÀ PHƯƠNG PHÁP BIẾN ĐỔI PQR

Trang 59

Dấu “=” xảy ra a b c

Ví dụ C.I.7: Cho , ,a b cChứng minh rằng

6 6 6 2 2 2 3 3 3 3 3 33 2a b c a b c a b b c c a

Chứng minh

Vì 6 6 6 26 6 6 2 2 23 3a b c a b c a b c abc

Và 3 3 33 3 3 3 3 32 2a b b c c a ab bc ca

Nên ta chỉ xét trường hợp , , 0a b c Theo bất đẳng thức Schur ta có

2 2 2 2 2 2 2 2 2 2 2 2 2 2 2

6 6 6 2 2 2 2 2 2 2 2 2 2 2 2 2 2 2

0

3

a a b a c b b c b a c c a c b

a b c a b c a b a b b c b c c a c a

Nên ta cần chứng minh

2 2 2 2 2 2 2 2 2 2 2 2 3 3 3 3 3 3

2 2 22 2 2 2 2 2

2

0

a b a b b c b c c a c a a b b c c a

a b a b b c b c c a c a

Bất đẳng thức trên luôn đúng nên ta có đpcm Dấu “=” xảy ra a b c hoặc , 0a b c cùng các hoán vị

C.II. Phương pháp biến đổi pqr C.II.1 . Đa thức đối xứng 3 biến

a) Khái niệm: Giả sử F(a,b,c) là đa thức với bộ 3 biến a,b,c Ta gọi F(a,b,c) là đa thức đối xứng 3 biến nếu F(a,b,c)=F(a’,b’,c’) trong đó (a’,b’,c’) là hoán vị tuỳ ý của (a,b,c).

b) Đa thức đối xứng Viete Định nghĩa: , ,p a b c q ab bc ca r abc được gọi là đa thức đối xứng

Viete Mệnh đề : Mọi đa thức đối xứng F(a,b,c) đều biểu diễn dưới dạng T(p,q,r). Các hằng đảng thức đáng nhớ

Page 60: Chuyên đề bất đẳng thức toán học

MỘT SỐ PHƯƠNG PHÁP CHỨNG MINH BẤT ĐẲNG THỨC BẤT ĐẲNG THỨC SCHUR VÀ PHƯƠNG PHÁP BIẾN ĐỔI PQR

Trang 60

2

2 2 2 2

3 3 3 3

2 2 2 2 2 2 2

3 3 3 3 3 3 3 2

4 4 4 4 2 2

2 2 2 2 2 2 2 2

23 3

23 3

3

4 2 4

2

a b b c c a pq r

a b a c b c b a c a c b p q

a b c p qa b c p pq ra b b c c a q pra b b c c a q pqr rab a b bc b c ca c a pq r

a b c p p q q pr

ab a b bc b c ca c a p q q pr

a

4 4 4 4 4 4 4 2 2 2 24 2 4b b c c a q pq r p r qr

C.II.2 . Xây dựng các bất đẳng thức của pqr

a) Xây dựng các bất đẳng thức từ bất đẳng thức Schur Với k=0

2 2 2

2 3a b c ab bc ca

p q

Với k=1

3 3 3

3

3

3

3 3 3 3 04 9 0

a b c abc ab a b bc b c ca c a

p pq r r pq rp pq r

Với k=2

4 4 4 2 2 2 2 2 2

4 2 2 2 2

4 2 2

4 2 4 2 05 4 6 0

a b c abc a b c ab a b bc b c ca c a

p p q q pr pr p q q prp p q q pr

b) Xây dựng từ các bất đẳng thức khác

2 2 2 2 2 3 2

2 2 2 2 2 3 2

3 4 2 2 3 3 3

1, 3 4, 3 4 7, 3 2 , 10, 9 42, 3 5, 3 4 8, 2 3 ,11, 2 9 73, 27 6, 3 4 9, 2 9 7 , 12, 6

p q p q pr q p q pr q q r pqrq pr pq qr p r pq p r qr q r pqrp r p q p q p r pq p r q pqr

CHỨNG MINH

2

2

2 2 2

1, 3

3 0

0

p q

a b c ab bc ca

a b b c c a

2

2

2 2 2

2, 3

3 0

0

q pr

ab bc ca a b c abc

ab bc bc ca ca ab

Page 61: Chuyên đề bất đẳng thức toán học

MỘT SỐ PHƯƠNG PHÁP CHỨNG MINH BẤT ĐẲNG THỨC BẤT ĐẲNG THỨC SCHUR VÀ PHƯƠNG PHÁP BIẾN ĐỔI PQR

Trang 61

3

3

3, 27

27 0

p r

a b c abc

(đúng theo bất đẳng thức Cauchy với 3 số)

2 2

2 2

2 2 2

4, 3 4

( ) 3 4

0

p q pr q

a b c ab bc ca abc a b c ab bc ca

ab a b bc b c ca c a

2 2

2 2

2 2 23 3 3

5, 3 4

3 4

0

pq qr p r

a b c ab bc ca abc ab bc ca abc a b c

a b c b c a c a b

4 2 2

2 2

6, 3 4

3 0

p q p q

p q p q

(luôn đúng do 2 3p q )

2 2

2 2 2

2 2 22 2 2

7, 3 2

3 4 2 3 0

0

p q pr q

p q pr q q pr

ab c a b bc a b c ca b c a

2 2

2 2 2

2 2 23 3 3

8, 2 3

3 4 2 3 0

0

pq p r qr

pq qr p r r p q

abc c a b abc b c a abc a b c

3

3

9, 2 9 7

2 4 9 9 0

p r pq

p pq r pq r

(đúng theo bất đẳng thức Schur và bất đẳng thức Cauchy)

3 2

3 2 2 2

10, 9 4

9 4 0

0

q r pqr

ab bc ca a b c abc a b c ab bc ca

ab ab bc ab ca bc bc ca bc ab ca ca ab ca bc

(luôn đúng theo bất đẳng thức Schur)

3 2

3 2

11, 2 9 7

2 4 9 9 0

q r pqr

q pqr r r pq r

3 3

2 2

12, 6

3 3 0

p r q pqr

pr p q q q pr

C.II.3 . Ví dụ

Ví dụ C.II.1: Cho , , 0a b c thoả mãn 4ab bc ca abc . Chứng minh rằng

2 2 23 10a b c abc

Page 62: Chuyên đề bất đẳng thức toán học

MỘT SỐ PHƯƠNG PHÁP CHỨNG MINH BẤT ĐẲNG THỨC BẤT ĐẲNG THỨC SCHUR VÀ PHƯƠNG PHÁP BIẾN ĐỔI PQR

Trang 62

Chứng minh Ta có

44

ab bc ca abcq r

2 2 2

2

2

3 10

3 6 103 7 6 0(1)

a b c abc

p q rp q

Tới đây ta có nhiều cách làm với nhiều ý tưởng khác nhau nhưng chắc hẳn đa số sẽ nghĩ tới việc làm giảm số biến bằng cách sử dụng bất đẳng Schur hoặc sử dụng một trong các bất đẳng thức liên hệ giữa p và q. Sau đây tôi xin giới thiệu 2 cách trong ý tưởng làm giảm số biến.

Cách 1: Sử dụng bất đẳng thức Schur Theo bất đẳng thức Schur bậc 1 ta có

3

3

3

9 49 4 4

36 (2)4 9

p r pqp q pq

pqp

Từ (1),(2) ta cần chứng minh

32

3 2

2

363 7. 6 04 9

5 27 24 306 0

3 5 42 102 0

3

ppp

p p p

p p p

p

Mặt khác ta giả sử 3p khi đó ta có 3

2

127

33

4

pr

pq

q r

vô lý 3p

Nên bất đẳng thức trên được chứng minh Đẳng thức xảy ra 1a b c

Cách 2:Ta có 2

2

3

3

p qpq

Nên ta cần chứng minh

Page 63: Chuyên đề bất đẳng thức toán học

MỘT SỐ PHƯƠNG PHÁP CHỨNG MINH BẤT ĐẲNG THỨC BẤT ĐẲNG THỨC SCHUR VÀ PHƯƠNG PHÁP BIẾN ĐỔI PQR

Trang 63

22 18 03

pp

Tới đây làm tương tự như cách 1 ta có đpcm

Qua ví dụ trên các bạn hẳn đã nhận thấy rằng cùng 1 phương pháp và cùng một ý tưởng nhưng với cách lựa chọn việc sử dụng bất đẳng thức khác nhau sẽ làm cho việc chứng minh sau khi rút biến trở nên dễ dàng hơn

Ví dụ C.II.2: Cho , , 0a b c thoả mãn 1abc Chứng minh rằng

3 3 331 1 11 1 1 1a b ca b c

(*)

Chứng minh Ta có

33 3 3 3 3 3 3 3 3 3 3 3

33 3

3 2

(*) 1

6 8 1

9 6 3 3 0(1)

ab bc caa b c a b c a b c a b cabc

p pq q q

p pq q q

Do 1 , 3r p q Nếu 3p q ta có

3 2(1) 3 9 9 6 12p q q pq q Theo bất đẳng thức Cauchy ta có

3 23 9 9p q q pq nên ta cần chứng minh 9 9 6 12

3 4pq pq q

pq q

Vì 23 3 4p q pq q q (do 3q ) Nếu 3q p theo bất đẳng thức Schur bậc 1 ta có

3 9 4p pq nên ta cần chứng minh 2 23 3 2

3q q q

q

(luôn đúng do r=1) Vậy bất đẳng thức được chứng minh Đẳng thức xảy ra 1a b c

Ví dụ C.II.3: Cho , , 0a b c thoả mãn 1ab bc ca CMR 1 1 1 3(*)ab bc caa b b c c a

Chứng minh Ta có

Page 64: Chuyên đề bất đẳng thức toán học

MỘT SỐ PHƯƠNG PHÁP CHỨNG MINH BẤT ĐẲNG THỨC BẤT ĐẲNG THỨC SCHUR VÀ PHƯƠNG PHÁP BIẾN ĐỔI PQR

Trang 64

2

2 2 2

2

2

(*) 1 3

1 1 3

4 3

2 4 3 33 3 2 01 2 3 0(1)

cyc

cyc

ab c a c b a b b c c a

ab c a b b c c a

a b c abc a b c a b b c c a

p q pr pq rp p r prp p r p

Nếu 2p ta có (1)luôn đúng Nếu 2p theo bất đẳng thức Schur bậc 1 ta có

3

3

4 9 03

9

p pq rp pr

nên ta cần chứng minh

3 2

2 2 31 2 0

92 5 3 9 0(2)

p p p pp p

p p p p

Vì 2p và 2

3 2 3 2 3 275 3 9 4 02 4

p p p p p p

Nên (2) luôn đúng đpcm Dấu “=” xảy ra 1, 0a b c cùng các hoán vị

Ví dụ C.II.4: Cho , , 0a b c và 0a b b c c a CMR

2 2 2

1 1 1 9 (*)4

ab bc caa b b c c a

Chứng minh Ta có

2 2 2

2 2 2 2 2 2 2 2 2

2

2 2 2

1 1 1 94

4 9

4 2

9

ab bc caa b b c c a

ab bc ca b c c a a b b c a b a c a b b c c a

ab bc ca a b a c b c b a c a c a a b c a b b c c a

a b b c c a

Theo pqr ta cần chứng minh

Page 65: Chuyên đề bất đẳng thức toán học

MỘT SỐ PHƯƠNG PHÁP CHỨNG MINH BẤT ĐẲNG THỨC BẤT ĐẲNG THỨC SCHUR VÀ PHƯƠNG PHÁP BIẾN ĐỔI PQR

Trang 65

22

4 2 2 3 2

3 4 2 2

4 4 9

4 17 4 34 9 0

4 9 5 4 6 9 0(1)

q p q p pq r pq r

p q p q q pqr r

pq p pq r q p p q q pr r pq r

Vì (1) luôn đúng nên ta có đpcm Dấu “=” xảy ra a b c hoặc , 0a b c cùng các hoán vị

Như ta đã thấy ở 3 ví dụ đầu khi đề bài cho các điều kiện rằng buộc giữa các biến p,q,r thì việc biến đổi, việc chứng minh cũng như định hướng cách làm khá đơn giản tuy nhiên ở ví dụ 4 khi không có điều kiện thì việc biến đổi trở nên khó khăn hơn.Để giải quyết vấn đề này chúng ra sử dụng phương pháp chuẩn hoá để có điều kiện giữa các biến giúp việc chứng minh trở nên dễ dàng hơn. Sau đây là một vài ví dụ về phương pháp biến đổi pqr kết hợp chuẩn hoá.

Ví dụ C.II.5: Cho các số thực không âm a,b,c CMR 4 4 4 2 2 2 2 2 2 2 2 2 (*)a b c ab bc ca a b c a b b c c a

Chứng minh Do (*) là bất đăng thức thuần nhất bậc 6 nên không mất tính tổng quát ta chuẩn hoá

1ab bc ca Khi đó ta có

4 4 4 2 2 2 2 2 2 2 2 2

4 2 2 2 2

4 2 2 3

4 2 3

4 2 2

4 2 4 2 2

4 2 4 2 2 45 2 4 0

5 4 6 2 3 0(**)

a b c ab bc ca a b c a b b c c a

p p q q pr q p q q pr

p p pr p p r prp p p r

p p pr pr p

Mà theo bất đẳng thức Schur ta có 4 2 2

4 2

5 4 6 05 6 4 0

p p q q prp p pr

Và 2 3 3p q nên (**) luôn đúng ta có đpcm Dấu “=” xảy ra a b c hoặc , 0a b c cùng các hoán vị Ví dụ ***:Cho , , 0a b c Chứng minh rằng

3 3 3

4.5(*)

a b c ab bc caa b cb c a c a b a b c

Chứng minh Do bất đẳng thức cần chứng minh là bất đẳng thức thuần nhất bậc nên ta chuẩn hoá

1a b c hay 1p Khi đó ta có 1 2 3 4(*) 5

1 3 3q r q

q r q r

Mặt khác ta lại có: 1 2 3 4 1 3 3 41

1 3 3 1 3 3q r q q r qVT

q r q r q q r

(do 0r abc )

Page 66: Chuyên đề bất đẳng thức toán học

MỘT SỐ PHƯƠNG PHÁP CHỨNG MINH BẤT ĐẲNG THỨC BẤT ĐẲNG THỨC SCHUR VÀ PHƯƠNG PHÁP BIẾN ĐỔI PQR

Trang 66

Áp dụng bất đẳng thức Cauchy ta có

1 3 3 41 3 3 4 2 41 3 3 1 3 3

5

q r qq r qq q r q q r

VT

đpcm

Dấu “=” xảy ra 3 5 3 5, , , ,0 ; , ,02 2

a b c x x x x

cùng các hoán vị (với x là số

dương bất kì)

Ví dụ C.II.6: Cho , , 0a b c Chứng minh rằng

2 2 2

4 4 4 6(*)ab bc ca bc ca ab ca ab bca bc b ac c ab

Chứng minh Không mất tính tổng quát chuẩn hoá q=3. Khi đó ta có

2 2 2

2 2 2

2 2 2 2 2

2 2 3 3 2 2 3 3 3 2 2 2

2 2 2 3 3 3 3 3

1 1 1(*) 3 6

1 1 1 2

1 2

2 3

2 2

cyc

cyc cyc cyc

cyc

bc ca aba bc b ca c ab

bc ca aba bc b ca c ab

bc b ca c ab a bc b ca c ab

b c b c ab a b abc a b c a b c abc a b c

a b c b c abc a b c

(1)

Theo pqr ta có

2 2 2 3 2 2 3

2 3 3 2

2 2

(1) 2 2 3 3 3 2 3 3

4 2 3 3 3 3

3 36 4 7 0(2)

q pr p q q pr pr q pqr r r r p pq r

r r p pq r q pqr r

p r pr

Mặt khác ta có 2 2 23 4 3 36p q pr q p q pr

9 3

4 03

pq r p rpr r

Nên ta có

2 8(2) 3 3 36 4 03 3p prp pr r r

Bất đẳng thức trên luôn đúng nên ta có đpcm Dấu “=” xảy ra , 0a b c cùng các hoán vị

Page 67: Chuyên đề bất đẳng thức toán học

MỘT SỐ PHƯƠNG PHÁP CHỨNG MINH BẤT ĐẲNG THỨC Phương pháp SS (Schur–S.O.S)

Trang 67

D Phương pháp SS (Schur–S.O.S)

D.I. Giới thiệu Như đã biết mọi bất đẳng thức 3 biến( đối xứng hoặc hoán vị) đều có thể biến đổi được về dạng tiêu chuẩn của S.O.S: 2 2 2( ) ( ) ( ) 0S b c S c a S a ba b c . Tuy nhiên việc biến đổi

về dạng tiêu chuẩn này thường không dễ dàng( đặc biệt là với những bài bất đẳng thức 3 biến hoán vị vòng quanh). Hơn nũa khi làm bất đẳng thức bằng phương pháp S.O.S thường phải xét 2 trường hợp( &a b c c b a ). Đây là điêu khá bất tiện. Phương pháp SS ra đời cũng từ những bất tiện này gây ra.SS( Schur - SOS ) là phương pháp đưa bất đẳng thức 3 biến thành dạng:

2M a b N c a c b

trong đó &M N là 2 biểu thức đối xứng với a và b . Như vậy chỉ cần giả sử max (min) , ,c a b c và chứng minh , 0M N thì bất đẳng thức đã được chứng minh. Cũng

như S.O.S thì phương pháp SS cũng có những khai triển rất quan trọng:

22 2 2 a b c ab bc ca a b c a c b

23 3 3 3a b c abc a b c a b a b c c a c b

26 2ab a b bc b c ca c a abc c a b a b c a c b

22 2 2 3a b b c c a abc c a b b c a c b

2 24 2

cyc cyca abc a a b c a b ab a c b c c a c b

23 2 2cyc cyc

a b c abc a a c b c a b ab ac bc c a c b

23 2

sym cyca b abc a ca cb a b a ac c a c b

21 13a b c a b c a c bb c a ab ac

226cyc

b c a ba b c a c ba ab abc

Phần

D

Page 68: Chuyên đề bất đẳng thức toán học

MỘT SỐ PHƯƠNG PHÁP CHỨNG MINH BẤT ĐẲNG THỨC Phương pháp SS (Schur–S.O.S)

Trang 68

23 1 22 2cyc

a a b ca b c a c bb c a c b c a b b c a c

222 1 13

sym

c a c b k k k a k b kck a ba kba kc ak c bk c a kb b ka c kb

2 2 2

2

2 2 22

2

a b c a b cb c a c a b

a b c a b ca b c a b c a c ba c b c a b b c c a

Nhìn chung, tư tưởng của SS có phần khá giống với S.O.S, còn các phép biến đổi của SS thường khó hơn S.O.S, tuy nhiên các hướng làm sau biến đổi đa dạng và đơn giản hơn S.O.S. Chính vì vậy chúng ta cần sử dụng linh hoạt và hợp lí hai phương pháp này.

D.II. Phương pháp SS với các bất đẳng thức đạt cực trị tại tâm

Ví dụ D.II.1: Cho , , 0a b c Chứng minh rằng 2 2 24

2a b cb c a c a b

a b c ab bc ca

Chứng minh Cách 1: Phương pháp SS

Bất đẳng thức cần chứng minh tương đương với:

2 2 2

22

2

2

6 4 1

2 4 4

2 4 4 0

2

a b cb c a c a ba b c ab bc ca

a b c a c ba ba b c a c bab abc ab bc ca ab bc ca

a ba b c a c bab ab bc ca abc ab bc ca

bc ca ab a b ab bc ca b

ab ab bc ca

2 22

22

40

220

20 (1)

a abcc a c b

abc ab bc ca

a b ab c ab a bbc ca aba b c a c b

ab ab bc ca abc ab bc ca

bc ca ab a b c ab a ba b c a c b

ab ab bc ca abc ab bc clllllll

a

Không mất

tính tổng quất giả sử max , ,c a b c .Dễ dàng chứng minh được bất đẳng thức (1) đúng.

Cách 2: Phương pháp S.O.S

Page 69: Chuyên đề bất đẳng thức toán học

MỘT SỐ PHƯƠNG PHÁP CHỨNG MINH BẤT ĐẲNG THỨC Phương pháp SS (Schur–S.O.S)

Trang 69

2 2 2

2 2 2

42

2 4 1cyc

a b cb c a c a ba b c ab bc ca

a b a b cb a ab bc ca

2 2

2

2

22 2

2 2 2

2

1 2 0

2 0

0

( ) ( ) ( ) 0

cyc cyc

cyc

cyc

cyc

a b a bab ab bc ca

a bab ab bc ca

c ab bc ca abc a b

bc ac abc a b

S b c S c a S a ba b c

trong đó:

2 2

2 2

2 2

S ba ca abcaS cb ab abcbS bc ac abcc

Giả sử a b c ( trường hợp a c b ta làm tương tự).Theo điều kiện 2: a b c và , , 0S S S S Sb b a c b thì bất đẳng thức đã được chứng minh).

Ví dụ D.II.2: Cho , , 0a b c Chứng minh rằng

2 2 2

2 2 2

a bc b ac c ab a b cb c a c a bb c c a a b

Chứng minh Bất đẳng thức đã cho tương đương với:

2 2 2

2 2 2 2 2 2

2

2

2

2 2 2

0

0

0

cyc

cyc

a bc b ac c ab ab ac ba bc ca cbb c c a a b b c c a a b

a bc ab acb c

a b a cb c

c a c b a c b ca ba b b c c a

Page 70: Chuyên đề bất đẳng thức toán học

MỘT SỐ PHƯƠNG PHÁP CHỨNG MINH BẤT ĐẲNG THỨC Phương pháp SS (Schur–S.O.S)

Trang 70

2 2 2 2

2 2 2

3 2 2 3 3 2 2 3

2 2 2

0

0

a c a c b c b cc a c ba b

a b b c c a

a a c ac c b b c bc cc a c ba b

a b b c c a

3 3 2 2 2

2 2 2

2 2 22

2 2 2

0

0 (1)

a b c a b c a bc a c ba b

a b b c c a

c a c b a ab b ca cb ca ba b b

ac

ac a

a a

Không mất tính tổng quát giả sử min , ,c a b c . Dễ thấy (1) đúng.

Vậy bất đẳng thức đã được chứng minh. Dấu bằng xảy ra khi a b c

Ví dụ D.II.3: Cho , , 0a b c Chứng minh rằng

2

2 3 3

2 113sym

a b abca b cb c

Chứng minh Bất đẳng thức đã cho tương đương với:

2

2 3 3 3

2 23 03sym

a b abca b cb c

Mặt khác:

2 2

22 2 2 2 2

2 2 23

sym

a b a b c a b c a b ca b c a c b

b c a b b c a b b c

(1)

2

3 3 3 3 3 3

22 23 3

a b cabc a b c a c ba b c a b c

(2)

Từ (1)&(2) ta có:

2

2 3 3 3

22

2 2 3 3 3

2 2 3 3 3

2 2 2 23 3 3

2 2

2 23 03

2 23

2 2 20

3

3 2 2

3 2 2 2

sym

a b abca b cb c

a b c a b ca b

a b ca b b c

a b c a b c a b cc a c b

a b ca b a c

a b c a b c a b c a b b c a b

a b c a b c a b c a b a c

0c a c b Không mất tính tổng quát giả sử max , ,c a b c

Page 71: Chuyên đề bất đẳng thức toán học

MỘT SỐ PHƯƠNG PHÁP CHỨNG MINH BẤT ĐẲNG THỨC Phương pháp SS (Schur–S.O.S)

Trang 71

Ta cần chứng minh:

2 2 23 3 3

2 23 3 3

3 2 2 0

3 2 2 2 0

a b c a b c a b c a b b c

a b c a b c a b c a b c a b a c

Thật vậy:

2 3 3 3

2 2 2

3 2

4

a b c a b c

a c b c a b c a b c

2 2 2

2 2 2 2

2

2 2

2 2

2 2 23 3 3

2 2

2 2

2

2

2

3 2 2 2 0

3 2 2 0

3 2 2 2 0

a b c a b b c a b c

a b c a b b c a b c b

a b c a b b c ab ac bc b

a b c a b b c

a b c a b c a b c a b a c

a b c a b c a b c a b b c

a b c a b c a b c a b a c

D.III. Phương pháp SS với các bất đẳng thức cực trị không đạt tại tâm Phương pháp SOS và SS tỏ ra khá hiệu quả với các bất đẳng thức 3 biến không chứa

căn, nhưng một yếu tố kiên quyết để đưa về dạng chuẩn của phương pháp là bất đẳng thức phải có dấu bằng đạt tại tâm. Tuy nhiên với những bất đẳng thức dấu bằng không xảy ra tại tâm, chúng ta cũng vẫn có thể áp dụng phương pháp SS để chứng minh. Các bài toán sau đây sẽ minh họa cho kỹ thuật này.

Ví dụ D.III.1: Cho , , 0

3a b ca b c

. Chứng minh: 2 2 2 274

a b c b c a c a b (1)

Chứng minh

32 2 2

32 2 2

(1)4

4 (2)

a b ca b c b c a c a b

a b c b c a c a b a b c

Đẳng thức không xảy ra tại tâm mà tại 3 , 02

a b c và các hoán vị nên nếu ta giữ nguyên

bất đẳng thức như trên thì khó có thể biến đổi về dạng chuẩn của phương pháp SOS và SS. Chúng ta sẽ xuất hiện ý tưởng cộng thêm một biểu thức không âm vào vế trái của bất đẳng thức (2) để có thêm điều kiện đẳng thức xảy ra tại tâm 1a b c nhưng vẫn phải bảo toán

điều kiện đẳng thức cũng xảy ra tại 3 , 02

a b c . Đại lượng cộng thêm đó là kabc. Cho a

= b = c = 1 ta sẽ được k = 3. Giả sử c = min (a , b, c) ta sẽ chứng minh BĐT mạnh hơn

Page 72: Chuyên đề bất đẳng thức toán học

MỘT SỐ PHƯƠNG PHÁP CHỨNG MINH BẤT ĐẲNG THỨC Phương pháp SS (Schur–S.O.S)

Trang 72

3 2 2 24 3a b c a b c b c a c a b abc (3)

3 3 3 2 2 2 2 2 2 2 2 2

2

3 6 4 3

0

a b c a b ab a c ac b c bc abc a b c b c a c a b abc

a b c a b c a c b c

Điều này đúng do c = min (a,b,c)

Bước tinh tế nhất trong lời giải trên chính là ý tưởng chuyển từ BĐT (2) về BĐT (3). Công việc tưởng chừng làm cho BĐT chặt hơn lại giúp tạo BĐT cực trị đạt tại tâm để có thể chứng minh bằng phương pháp SS hay SOS. Đối với những bất đẳng thức có dấu bẳng xảy ra tại biên, đại lượng công thêm vào thường có dang k.abc.f (a, b, c) để đảm bảo cả dấu bằng BĐT ban đầu vẫn xảy ra tại biên và dấu bằng BĐT mới có xảy ra tại tâm. Từ đó chúng ta có thể thấy, việc tìm f (a, b, c) thích hợp chính là nét tinh tế, cũng như khó nhất của việc biến đổi này.

Ví dụ D.III.2: Cho , , 0

2a b ca b c

. Chứng minh rằng 2 2 2 2 2 2 1a b b c c a (1)

Chứng minh 4 2 2 2 2 2 2(1) 16 8a b c a b b c c a abc a b c

Ta sẽ chứng minh

4 2 2 2 2 2 2

4 4 4 2 2 2 2 2 2 3 3 3

2 2 2 2 2 2

4 2 2 3

16 11

( ) 6 4 ( ) 12

16 11

10 4 2cyc cyc cyc cyc cyc cyc

a b c a b b c c a abc a b c

a b c a b b c c a a b c b a c c a b abc a b c

a b b c c a abc a b c

a abc a a b abc a a b c abc a

2 2 2 2

2

0

10 4

10 8 4 4 0

. . 0

a b a b c c a c b c

c a c b ab a c b c ab ab ac bc

M a b N c a c b

Với 2 2 2

2

6 4 4 55 5

M a b ab ac bc cN ac bc c

Dễ thấy với min , ,c a b c thì bất đẳng thức được chứng minh

Ví dụ D.III.3: Chứng minh rằng: 2 2 2 2

a b c b c a c a ba bc b ca c ab

, , 0a b c

Chứng minh

Ta sẽ chứng minh

2 2 2

2 2 2 2 2 2

82a b c b c a c a b a b ca bc b ca c ab a bc b ca c ab

Page 73: Chuyên đề bất đẳng thức toán học

MỘT SỐ PHƯƠNG PHÁP CHỨNG MINH BẤT ĐẲNG THỨC Phương pháp SS (Schur–S.O.S)

Trang 73

2 2 2 2 2 2 2 2

4 2 2 2 3 3 3 2 2 2

2 8

3 2 12sym

sym sym sym sym

a b c b ac c ab a bc b ac c ab a b c

a b c abc a b c b c abc a a b c

2 22 24 2 2 0ac bc ab c ab c ac bc a b abc a b a c b c

Giả sử min , ,c a b c bất đẳng thức trên đúng, ta có điều phải chứng minh

Ví dụ D.III.4: Chứng minh 2 2 2 2 2 2

4 1 1 1 , , 05

a b c a b cb c c a a b b c c a a b

Chứng minh Ta sẽ chứng minh bất đẳng thức mạnh hơn:

2 2 2

2 2 2 2 2 2 2 2 2 2 2 2

4 1 1 1 365 5

a b c a b cb c c a a b b c c a a b a b b c c a a b c

2 2 2 2

2 2 2 2 2 2 2 2 2 2

12 365 5sym sym sym

a b c a a a b cb c b c b c a b b c c a

Ta có thể chứng minh được các bất đẳng thức sau: 2

2 2

4 45 5sym sym

a ab c b c

(1)

2 2 2 2

2 2 2 2 2 2 2 2

1 4 1 .25 5sym

a a b cb c a b b c c a

(2)

2 2 2

2 2 2 2 2 2 2 2

82sym

a b c a b cb c a b b c c a

(3)

Cộng 2 vế của (1), (2) và (3) ta được điều phải chứng minh.

Phương pháp S.S tỏ ra khá hiệu quả ngay cả trong các bất đẳng thức dấu = không xảy ra tại tâm. Tuy nhiên, một nhược điểm của phương pháp này đó là việc biến đổi khá phức tạp do không mang tính đối xứng như phương pháp S.O.S Việc biến đổi thành dạng chuẩn của phương pháp là một kỹ năng hết sức quan trọng. Bên cạnh đó, việc chứng minh M và N dương trong các bài toán cũng không hề đơn giản, thường yêu cầu những chứng minh khá dài, có thể dùng đạo hàm… Tuy nhiên, xét về tổng thể, phương pháp S.S có đường lối rõ ràng và chứng minh được rất nhiều bất đẳng thức khó. Vì vậy, đây là một phương pháp mạnh bên cạnh các phương pháp khác, làm phong phú thêm các cách chứng minh bất đẳng thức hiện đại.

Page 74: Chuyên đề bất đẳng thức toán học

Trang 74

LỜI KẾT ậy là sau 1 tháng thực hiện, chuyên đề của chúng tôi đã hoàn thành. Mọi người đều rất tâm đắc với kết quả làm việc này của

mình. Một lần nữa, chúng em xin gửi lời biết ơn sâu sắc đến thầy giáo Đoàn Thái Sơn và những người đã giúp chúng em hoàn thành chuyên đề này.

Như chúng tôi đã nói, bất đẳng thức là một trong những mảng kiến thức khó và rất đa dạng của toán học. Các phương pháp trình bày trong chuyên đề của chúng tôi không phải là những phương pháp mẫu, đó chỉ là một trong những hướng đi trong vô vàn cách tiếp cận của các bài toán bất đẳng thức. Chúng tôi hy vọng sau khi đọc xong chuyên đề này, các bạn sẽ vận dụng một cách sáng tạo những phương pháp được trình bày vào giải toán để tự kiểm chứng sức mạnh của từng phương pháp và đặc biệt là sẽ tìm tòi khám phá ra những phương pháp mới độc đáo cho riêng mình.

Trần Tiến Đạt và các cộng sự Lưu Trung Kiên – Nguyễn Hồng Đức – Đỗ Thị Hồng Vân

V

Page 75: Chuyên đề bất đẳng thức toán học

Một số phương pháp chứng minh bất đẳng thức

Phương pháp S.O.S

Phương pháp dồn biến

Phương pháp P.Q.R

Phương pháp S.S

----------------------------------------

Trần Tiến Đạt

Lưu Trung Kiên

Nguyễn Hồng Đức

Đỗ Thị Hồng Vân

Lớp 10 Toán trường THPT chuyên Trần Phú – Hải Phòng

Tháng 5/2009